1B - General-Purpose Financial Statements: For-Profit Business Entities Flashcards

1
Q

Redwood Co.’s financial statements had the following information at year-end:

Cash $ 60,000
Accounts receivable 180,000
Allowance for uncollectible accounts 8,000
Inventory 240,000
Trading debt securities 90,000
Prepaid rent 18,000
Current liabilities 400,000
Long-term debt 220,000
What was Redwood’s quick ratio?
0.81 to 1

  1. 83 to 1
  2. 94 to 1
  3. 46 to 1
A

.81

Quick ratio = Current assets (excluding Inventories and Prepaid assets) ÷ Current liabilities:

(Cash + Net accounts receivable + Trading debt securities) ÷ Current liabilities
($60,000 + ($180,000 - $8,000) + $90,000) ÷ $400,000 = 0.81 (rounded)
All of the assets listed are current. Inventory and prepaid rent are excluded from the quick ratio.

How well did you know this?
1
Not at all
2
3
4
5
Perfectly
2
Q

Abbott Co. is preparing its statement of cash flows for the year. Abbott’s cash disbursements during the year included the following: Payment of interest on bonds payable $500,000
Payment of dividends to stockholders 300,000
Payment to acquire 1,000 shares of Marks Co. common stock 100,000

What should Abbott report as total cash outflows for financing activities in its statement of cash flows?
$0
$300,000
$900,000
$800,000

A

300,000
Interest payments are included in operating activities. Cash payments to acquire equity instruments are included in investing activities. Only payments of dividends to stockholders are included in financing activities.I

How well did you know this?
1
Not at all
2
3
4
5
Perfectly
3
Q

The statement of cash flows (SCF) is presented in three primary sections: cash flows from ___ activities, cash flows from ___ activities, and cash flows from ___ activities (in that order).

A

Operating, Investing, Financing

How well did you know this?
1
Not at all
2
3
4
5
Perfectly
4
Q

Cash flows from operating activities: Cash flows from operating activities include those cash flows resulting from transactions included in the determination of net income, unless specifically classified as financing or investing activities. ***TRUE OR FALSE?***

A

TRUE

How well did you know this?
1
Not at all
2
3
4
5
Perfectly
5
Q

Cash inflows from operating activities generally include the following:

(a) Cash receipts from sales of ___ or services
(b) Cash receipts from ___ and ___ on investments in another enterprise
(c) All other cash receipts that are not classified as either investing or financing activities. **T/F**

A

goods
Interest and Dividends
True

How well did you know this?
1
Not at all
2
3
4
5
Perfectly
6
Q

Cash outflows classified as operating activities include the following:

(a) Cash payments to acquire materials for manufacture or goods for resale
(b) Cash payments to other suppliers and employees for goods and services
(c) Cash payments to governments for taxes, duties, other fees, or penalties
(d) Cash payments to lenders and other creditors for interest
(e) All other cash payments that are not classified as investing or financing activities

T/F

A

True

How well did you know this?
1
Not at all
2
3
4
5
Perfectly
7
Q

A public entity sells steel for use in construction. One of its customers accounts for 43% of sales, and another customer accounts for 40% of sales. What should the entity disclose in its annual financial statements about these two customers?

The financial condition of the two customers

The payment terms of accounts receivable due from each of the two customers

The names of the two customers

The amount of the entity’s revenue from each of the two customers

A

The amount of the entity’s revenue from each of the two customers.

If revenues from transactions with a single external customer amount to 10% or more of an entity’s revenues, the business must disclose that fact, the total amount of revenues from each such customer, and the identity of the segments reporting the revenues.

A major customer is a single customer, or a group of entities, known to a reporting enterprise to be under common control.

The identity, payment terms, and financial condition of the customers do not need to be disclosed.

How well did you know this?
1
Not at all
2
3
4
5
Perfectly
8
Q

If revenues from transactions with a single external customer amount to __% or more of an entity’s revenues, the business must disclose that fact, the total amount of revenues from each such customer, and the identity of the segments reporting the revenues.

A

10%

How well did you know this?
1
Not at all
2
3
4
5
Perfectly
9
Q

If revenues from transactions with a single external customer amount to 10% or more of an entity’s ___, the business must disclose that fact, the total ___of revenues from each such customer, and the identity of the ___ _____the revenues.

A

Revenues
Amount
Segment Reporting

How well did you know this?
1
Not at all
2
3
4
5
Perfectly
9
Q

If revenues from transactions with a single external customer amount to 10% or more of an entity’s ___, the business must disclose that fact, the total ___of revenues from each such customer, and the identity of the ___ _____the revenues.

A

Revenues
Amount
Segment Reporting

How well did you know this?
1
Not at all
2
3
4
5
Perfectly
10
Q

A ___is a functional or responsibility area within a business that can be reported upon separately.

A

Segment

How well did you know this?
1
Not at all
2
3
4
5
Perfectly
11
Q

Fara Co. reported bonds payable of $47,000 on December 31, 20X1, and $50,000 on December 31, 20X2. During 20X2, Fara issued $20,000 of bonds payable in exchange for equipment. There was no amortization of bond premium or discount during the year. What amount should Fara report in its 20X2 statement of cash flows for redemption of bonds payable?

$20,000

$23,000

$3,000

$17,000

A

$17,000

Using the basic accounting equation, Beginning balance + Additions - Deletions = Ending balance:

Bonds payable on 12/31/X1 (beginning inventory) $47,000
Plus bonds issued in 20X2 20,000
Subtotal 67,000
Less bonds payable on 12/31/X2 (ending inventory) 50,000
Bonds redeemed in 20X2 (presumably for cash) $17,000

How well did you know this?
1
Not at all
2
3
4
5
Perfectly
12
Q

Cash flows from ___ activities involve asset transactions other than cash (and cash equivalents) and those assets related directly to the determination of operating results (e.g., inventories, receivables).

A

Investing

How well did you know this?
1
Not at all
2
3
4
5
Perfectly
13
Q

Specifically, the following are types of cash inflows from investing activities:

(a) Cash receipts from collections or sales of ___made by the enterprise and of other debt instruments that are purchased by the enterprise(b) Cash receipts from sales of ___securities of other enterprises(c) Cash receipts from the sales of ___ ___and other productive assets

A

Loans

Equity

fixed assets

How well did you know this?
1
Not at all
2
3
4
5
Perfectly
14
Q

Cash flows from ___ activities involve debt and equity financing.

Cash inflows from financing activities include the following:

(a) Cash proceeds from issuing ___ instruments
(b) Cash proceeds from issuing ___, ____, ___, and other short- and long-term debt instruments

A

Financing

equity

bonds, mortgages notes

How well did you know this?
1
Not at all
2
3
4
5
Perfectly
15
Q

Which of the following statements is correct regarding reporting comprehensive income?

Comprehensive income must include all changes in stockholders’ equity for the period.

Comprehensive income is reported in the year-end statements but not in the interim statements.

A separate statement of comprehensive income is required.

Accumulated other comprehensive income is reported in the stockholders’ equity section of the balance sheet.

A

Accumulated other comprehensive income is reported in the stockholders’ equity section of the balance sheet.

FASB ASC 220-10-45-14 requires that accumulated other comprehensive income be reported in the stockholders’ equity section of the balance sheet:

How well did you know this?
1
Not at all
2
3
4
5
Perfectly
16
Q

Accumulated other comprehensive income is be reported in the stockholders’ equity section of the balance sheet. T/F

Accumulated other comprehensive income is be reported in the stockholders’ equity section of the balance sheet COMBINED with retained earnings? T/F

component of equity.”

A

True

FALSE - It is to be reported separately

How well did you know this?
1
Not at all
2
3
4
5
Perfectly
17
Q

When computing the amount of the gain or loss from discontinued operations:

the gain or loss on the disposal of the discontinued operation is not included in the computation of the total gain or loss.

the operating results of the operation being discontinued are not included in the computation of the total gain or loss.

both the operating results and the gain or loss on disposal are included in the computation of the total gain or loss.

the gain or loss on disposal of the discontinued operation is reported net of tax but the operating results are not.

A

both the operating results and the gain or loss on disposal are included in the computation of the total gain or loss.

The computation of the total gain or loss from discontinued operations is computed by combining the operating results (income or loss) from the point in time when the disposal qualifies as a discontinued operation until the disposal is complete, with the gain or loss on disposal. The net amount of the two components is reported net of tax after “income from continuing operations” in the income statement.

How well did you know this?
1
Not at all
2
3
4
5
Perfectly
18
Q

The computation of the total gain or loss from ___ ___ is computed by combining the operating results (income or loss) from the point in time when the disposal qualifies as a discontinued operation until the disposal is complete, with the gain or loss on disposal.

The net amount of the two components is reported net of tax after “income from continuing operations” in the income statement.

A

discontinued operations

How well did you know this?
1
Not at all
2
3
4
5
Perfectly
19
Q

Discontinued operations are presented in a separate section of the income statement, just below income from ___ ___.

To be reported as discontinued operations, a disposal must represent a ___ ___ . A ___ ____must have a major effect on the entity’s operations and financial results

A

Continuing operations

Strategic shift – strategic shift

How well did you know this?
1
Not at all
2
3
4
5
Perfectly
20
Q

If a business or nonprofit activity is acquired to be held for sale, its disposal is considered a disposal of a discontinued operation. The initial criteria for classification as “held for sale” include:

  1. management has committed to a ___to sell the component;
  2. the entity is available for ___ sale;
  3. an active program to sell has been ___;
  4. the sale is expected to be complete within ___ ___;
  5. the entity is being actively ___; or
  6. it is unlikely the plan to sell will be ___.
A

Plan

immediate

initiated

one year

marketed

withdrawn

How well did you know this?
1
Not at all
2
3
4
5
Perfectly
21
Q

The entity must qualitatively assess whether a strategic shift has occurred for it to be considered a discontinued operation. Examples of a strategic shift include:

  1. a sale of a product line that represents __% of the entity’s total revenues;
  2. a sale of a geographical area that represents __% of the entity’s total assets;
A

15%

20%

How well did you know this?
1
Not at all
2
3
4
5
Perfectly
22
Q

A 70%-owned subsidiary company declares and pays a cash dividend. What effect does the dividend have on the retained earnings and noncontrolling (minority) interest balances in the parent company’s consolidated balance sheet?

A decrease in retained earnings and no effect on noncontrolling interest

Decreases in both retained earnings and noncontrolling interest

No effect on retained earnings and a decrease in noncontrolling interest

No effect on either retained earnings or noncontrolling interest

A

No effect on retained earnings and a decrease in noncontrolling interest

The dividend will have no effect on consolidated retained earnings because consolidated retained earnings include only retained earnings of the parent company.

However, since the noncontrolling (minority) interest (in this case, 30%) is a percentage of the stockholder equity (including retained earnings) of the subsidiary, any reduction in subsidiary retained earnings (such as dividend declaration) will decrease noncontrolling (minority) interest.

How well did you know this?
1
Not at all
2
3
4
5
Perfectly
23
Q

A parent company may own less than 100% of the voting stock of a subsidiary. In such cases, there is a ___(minority) interest in the net assets of the subsidiary as well as a ___interest.

A

noncontrolling

controlling

How well did you know this?
1
Not at all
2
3
4
5
Perfectly
24
Q

The noncontrolling interest is reported in the ___statement of financial position within equity

A

consolidated

How well did you know this?
1
Not at all
2
3
4
5
Perfectly
25
Q

In its 20X7 income statement, Brinkley Company reported cost of goods sold of $625,000. Changes occurred in several balance sheet accounts as follows:

Inventory $79,000 increase
Accounts payable - suppliers 61,000 increase

What amount should Brinkley Company report as cash paid to suppliers in its 20X7 cash flow statement, prepared under the direct method?

$607,000

$485,000

$765,000

$643,000

A

Cost of goods sold $625,000
Inventory increase 79,000
Purchases $704,000
Accounts payable increase (61,000)
Cash paid to suppliers $643,000

.

An increase in inventory during the period indicates that purchases were more than the cost of goods sold. Therefore, the current period’s amount of purchases is determined by adding the increase in inventory to cost of goods sold.

The ending accounts payable represents what is still owed to suppliers at the end of the current period for purchases. An increase in accounts payable during the current period indicates that suppliers were paid an amount of cash less than the amount of the current period’s purchases.

Therefore, subtracting the increase in accounts payable from purchases of the period yields the cash paid to suppliers in the current period.

How well did you know this?
1
Not at all
2
3
4
5
Perfectly
26
Q

The SCF presents net cash flows from operating, investing, and financing activities to reconcile the change in ___(and cash equivalents) for the period.

A

cash

How well did you know this?
1
Not at all
2
3
4
5
Perfectly
27
Q

In reporting cash flows from operating activities, enterprises are encouraged to use the ___method,

A

direct

How well did you know this?
1
Not at all
2
3
4
5
Perfectly
28
Q
  1. Information about ___-___investing and/or financing activities must be disclosed outside the main body of the SCF.
  2. Cash-flow-per-share information is REQUIRED to be presented. T/F
A
  1. non-cash
  2. False - it shall NOT be presented
How well did you know this?
1
Not at all
2
3
4
5
Perfectly
29
Q

(1) Cash collected from customers(2) Interest and dividends received(3) Other operating cash receipts(4) Cash paid to employees and other suppliers of goods and services(5) Interest paid(6) Income taxes paid(7) Other operating cash payments

This is an example of the INDIRECT method … T/F

A

False - this is the DIRECT method

How well did you know this?
1
Not at all
2
3
4
5
Perfectly
30
Q

Regardless of whether the direct or indirect method is used to determine cash flows from operating activities, the following items are required to be disclosed in the SCF or related notes:

(1) Amount of ___ ___paid during the period(2) Amount of ___ paid during the period(3) Reconciliation of ___ ___ and net cash flows from operating activities

A

Income Taxes

Interest

Net Income

How well did you know this?
1
Not at all
2
3
4
5
Perfectly
31
Q

Bay Manufacturing Co. purchased a 3-month U.S. Treasury bill. In preparing Bay’s statement of cash flows, this purchase would:

have no effect.

be treated as an outflow from investing activities.

be treated as an outflow from operating activities.

be treated as an outflow from financing activities.

A

have no effect.

FASB ASC 230-10-20 focuses on cash and cash equivalents. The following explanation is offered: “Examples of items commonly considered to be cash equivalents are Treasury bills, commercial paper, money market funds, and federal funds sold (for an enterprise with banking operations).”

Thus, the purchase of U.S. Treasury bills would have no effect on the statement of cash flows.

How well did you know this?
1
Not at all
2
3
4
5
Perfectly
32
Q

Rowe, Inc., owns 80% of Cowan Co.’s outstanding capital stock. On November 1, Rowe advanced $100,000 in cash to Cowan. What amount should be reported related to the advance in Rowe’s consolidated balance sheet as of December 31?

$0

$80,000

$100,000

$20,000

A

$0

All intercompany liabilities are eliminated in the consolidation process. The amounts are not included as assets or liabilities on the consolidated balance sheet.consolidated balance sheet.

How well did you know this?
1
Not at all
2
3
4
5
Perfectly
33
Q

Another type of eliminating entry the candidate should look for is that relating to year-end reciprocal balance sheet accounts. The following are examples of such accounts:

a. Accounts receivable or accounts payableb. Notes receivable or notes payablec. Advance to sub (parent) or advance from parent (sub)

A

yep! Just understand this.

How well did you know this?
1
Not at all
2
3
4
5
Perfectly
34
Q

Albany Co. has net income of $39,000, $17,000 of prior service costs related to amendments implemented in their pension plan, a gain of $8,100 on the effective portion of a cash flow hedge, and an impairment loss of $6,000 on an intangible asset. What amount is Albany’s comprehensive income?

$13,900

$47,100

$22,000

$30,100

A

$30,100

Comprehensive income includes net income and other comprehensive income (OCI). OCI includes pension-related items like the implementation of prior service costs and gains and losses on the effective portion of cash flow hedges as well as unrealized holding gains or losses on available-for-sale debt securities. The impairment loss ($6,000) is already included in net income.

Net income $39,000
Prior service costs related to a pension plan (17,000)
Gain on effective portion of cash flow hedge 8,100
$30,100
=======

Note: Amortization of prior service costs would be recognized in pension expense and included in net income.

How well did you know this?
1
Not at all
2
3
4
5
Perfectly
35
Q

Comprehensive income is defined in SFAC 6 as “the change in equity (net assets) of a business enterprise during a period from transactions and other events and circumstances from ___sources.

A

Non-owner

How well did you know this?
1
Not at all
2
3
4
5
Perfectly
36
Q

___ ___ ___ is defined as revenues, expenses, gains, and losses that under GAAP are included in comprehensive income but excluded from net income.

A

Other Comprehensive Income

How well did you know this?
1
Not at all
2
3
4
5
Perfectly
37
Q

Other comprehensive income (OCI) is defined as revenues, expenses, gains, and losses that under GAAP are included in comprehensive income but excluded from net income.

Here are some examples:

  1. Foreign currency ___adjustments
  2. Gains and losses on ___currency transactions
  3. Gains and losses on ___instrument
  4. Unrealized holding gains and losses on ___-___-___debt securities
  5. Gains or losses associated with ___or other ___benefits
A

translation

foreign

derivative

unrealized – available for sale

pension – postretirement

How well did you know this?
1
Not at all
2
3
4
5
Perfectly
38
Q

Which of the following describes how comprehensive income should be reported?

May be reported in a combined statement of income and comprehensive income or disclosed within a statement of stockholders’ equity; separate statements of comprehensive income are not permitted

Should not be reported in the financial statements but should only be disclosed in the footnotes

May be reported in a separate statement or in a combined statement of income and comprehensive income

Must be reported in a separate statement, as part of a complete set of financial statements

A

May be reported in a separate statement or in a combined statement of income and comprehensive income

An entity reporting comprehensive income in a single continuous financial statement shall present its components in two sections, net income and other comprehensive income.”

The financial statement should include a total net income amount , total other comprehensive income amount, and total comprehensive income.

How well did you know this?
1
Not at all
2
3
4
5
Perfectly
39
Q

An entity reporting comprehensive income in a sA company reports the following information for year 1:

Sale of equipment $20,000
Issuance of the company’s bonds 10,000
Dividends paid 5,000
Purchase of stock of another company 2,000
Purchase of U.S. Treasury note 2,000
Income taxes paid 2,000
Interest income received 500

What is the company’s net cash flow from financing activities?ingle continuous financial statement shall present its components in

$5,000

($9,000)

$15,000

$5,500.

A

$5,000

Cash Flows from Financing Activities:
Issuance of the company’s bonds $10,000
Dividends paid (5,000)
Net Cash Flows from Financing Activities $ 5,000

How well did you know this?
1
Not at all
2
3
4
5
Perfectly
40
Q

The following information was taken from the current year financial statements of Planet Corp.:

Accounts receivable, January 1 $ 21,600
Accounts receivable, December 31 30,400
Sales on account and cash sales 438,000
Uncollectible accounts 1,000

No accounts receivable were written off or recovered during the year. If the direct method is used in the current-year statement of cash flows, Planet should report cash collected from customers as:

A

$429,200.

The cash collected is $429,200, calculated as follows:

Sales $438,000
Increase in accounts receivable
($30,400 – $21,600) 8,800
Cash collected from customers $429,200

How well did you know this?
1
Not at all
2
3
4
5
Perfectly
41
Q

On January 1, 20X1, Prim, Inc., acquired all the outstanding common shares of Scarp, Inc., for cash equal to the book value of the stock. The carrying amounts of Scarp’s assets and liabilities approximated their fair values, except that the carrying amount of its building was more than fair value. The combination is accounted for as an acquisition. In preparing Prim’s 20X1 consolidated income statement, which of the following adjustments would be made?

Depreciation expense would be decreased and goodwill impairment would be assessed.

Depreciation expense would be decreased and goodwill impairment would not be assessed.

Depreciation expense would be increased and goodwill impairment would not be assessed.

Depreciation expense would be increased and goodwill impairment would be assessed.

A

Depreciation expense would be decreased and goodwill impairment would be assessed.

Regardless of the depreciation issue, FASB ASC 350-20-35-28 requires goodwill to be tested for impairment at least annually, as well as in the year of acquisition. Therefore, goodwill impairment must be assessed in this case.

How well did you know this?
1
Not at all
2
3
4
5
Perfectly
42
Q

___is the excess of the fair value of the consideration given over the fair value of the net identifiable assets acquired

A

Goodwill

How well did you know this?
1
Not at all
2
3
4
5
Perfectly
43
Q

Duke Co. reported cost of goods sold of $270,000 for 20X1. Additional information is as follows:

December 31__January 1
Inventory $60,000 $45,000
Accounts payable 26,000 39,000

If Duke uses the direct method, what amount should Duke report as cash paid to suppliers in its 20X1 statement of cash flows?

$272,000

$298,000
$268,000

$242,000

A

$298,000

Duke should report $298,000, calculated as follows:

Reported cost of goods sold for 20X1 $270,000
Add increase in inventory ($60,000 − $45,000) 15,000
Decrease in accounts payable ($39,000 − $26,000) 13,000
Cash paid to suppliers in 20X1 $298,000
========

How well did you know this?
1
Not at all
2
3
4
5
Perfectly
44
Q

Universe Co. issued 500,000 shares of common stock in the current year. Universe declared a 30% stock dividend. The market value was $50 per share, the par value was $10, and the average issue price was $30 per share. By what amount will Universe decrease stockholders’ equity for the dividend?

$0

$1,500,000

$4,500,000

$7,500,000

A

$0

Stock dividends are accounted for by reclassifying a portion of retained earnings as contributed capital. They do not reduce assets or increase liabilities. Therefore, total stockholders’ equity is not changed.

How well did you know this?
1
Not at all
2
3
4
5
Perfectly
45
Q

Stock dividends constitute income to the investor. T/F

A stock dividend takes nothing from the property of the corporation and adds nothing to the interests of the stockholders; that is, the corporation’s property is not diminished and the interests of the stockholders are not increased. T/F

The proportional interest of each shareholder changes. T/F

A

False - it does NOT constitute income to the investor

TRUE

FALSE - the share stays the SAME

How well did you know this?
1
Not at all
2
3
4
5
Perfectly
46
Q

___stock dividends are usually accounted for on the basis of the par or stated value of the stock rather than on the basis of fair value

A

Large

How well did you know this?
1
Not at all
2
3
4
5
Perfectly
47
Q

During January of the previous year, Doe Corp. agreed to sell the assets and product line of its Hart division. The sale on January 15 of the current year resulted in a gain on disposal of $900,000. Not con­sidering any impairment losses, Hart’s operating losses were $600,000 for the previous year and $50,000 for the current-year period January 1 through January 15. Disregarding income taxes, what amount of net gain (loss) should be reported in Doe’s comparative current and previous years’ income statements?

Current year, $250,000; Previous year, $0

Current year, $0; Previous year, $250,000

Current year, $900,000; Previous year, $(650,000)

Current year, $850,000; Previous year, $(600,000)

A

Current year, $850,000; Previous year, $(600,000)

The sale of a division would be a discontinued operation since its disposition represents a strategic shift. The discontinued operation would be recorded in the year the sale occurred.

Previous Current
Net loss from continuing operations $(600,000) $(50,000)
Gain on sale of discontinued operations 900,000
Net income $(600,000) $850,000

How well did you know this?
1
Not at all
2
3
4
5
Perfectly
48
Q

A discontinued operation must be presented separately, either on the balance sheet or in the footnotes, in the period it is classified as held for sale and for all prior periods presented. T/F

The assets and liabilities of a disposal group must be presented separately in the asset and liability sections, respectively. T/F

A gain or loss recognized for a long-lived asset classified as held for sale that is not a component of an entity must be included in income from continuing operations. T/F

A

True

True

True

How well did you know this?
1
Not at all
2
3
4
5
Perfectly
49
Q

Which of the following should be disclosed as supplemental information in the statement of cash flows?

Both cash flow per share and conversion of debt to equity

Cash flow per share

Conversion of debt to equity

Neither cash flow per share nor conversion of debt to equity

A

Conversion of debt to equity

Accounting guidance states very specifically that “financial statements shall not report an amount of cash flow per share.”

Also, “information about all investing and financing activities of an enterprise during a period that affects recognized assets or liabilities shall be reported in related disclosures.”

Converting debt to equity is cited as an example of affecting recognized liabilities

How well did you know this?
1
Not at all
2
3
4
5
Perfectly
50
Q

The following changes in Vel Corp.’s account balances occurred during 20X1:

Increase
Assets $89,000
Liabilities 27,000
Capital stock 60,000
Additional paid-in capital 6,000

Except for a $13,000 dividend payment and the year’s earnings, there were no changes in retained earnings for 20X1. What was Vel’s net income for 20X1?

$9,000

$13,000

$17,000

$4,000

A

$9,000

Increases in assets must equal increases in liabilities and equity (specifically increase in retained earnings in equity): Assets = Liabilities + Equity.

Increase in Assets $89,000
Increase in Liabilities (27,000)
Increase in stockholder’s equity $62,000
Add back: Dividend Payment 13,000
Increase in stockholders’ equity
BEFORE dividends $75,000
Less increase-new capital stock issued:
Capital Stock $60,000
Additional Paid-in Capital 6,000__66,000
20X1 Net Income $ 9,000

How well did you know this?
1
Not at all
2
3
4
5
Perfectly
50
Q

The following changes in Vel Corp.’s account balances occurred during 20X1:

Increase
Assets $89,000
Liabilities 27,000
Capital stock 60,000
Additional paid-in capital 6,000

Except for a $13,000 dividend payment and the year’s earnings, there were no changes in retained earnings for 20X1. What was Vel’s net income for 20X1?

$9,000

$13,000

$17,000

$4,000

A

$9,000

Increases in assets must equal increases in liabilities and equity (specifically increase in retained earnings in equity): Assets = Liabilities + Equity.

Increase in Assets $89,000
Increase in Liabilities (27,000)
Increase in stockholder’s equity $62,000
Add back: Dividend Payment 13,000
Increase in stockholders’ equity
BEFORE dividends $75,000
Less increase-new capital stock issued:
Capital Stock $60,000
Additional Paid-in Capital 6,000__66,000
20X1 Net Income $ 9,000

How well did you know this?
1
Not at all
2
3
4
5
Perfectly
51
Q

The income statement may be presented in either of two formats—single step or multiple step. T/F

A

True

How well did you know this?
1
Not at all
2
3
4
5
Perfectly
52
Q

The single-step income statement is a simple and relatively straightforward presentation whereby all revenues and gains are combined at the top of the statement. T/F

A

True

How well did you know this?
1
Not at all
2
3
4
5
Perfectly
53
Q

Under the multiple-step income statement, a distinction is made between operating and nonoperating items. T/F

Example: Operating revenues and non-operating revenues /// operating expenses and non-operating expenses

A

True

How well did you know this?
1
Not at all
2
3
4
5
Perfectly
54
Q

On September 29, 20X1, Wall Co. paid $860,000 for all the issued and outstanding common stock of Hart Corp. On that date, the carrying amounts of Hart’s recorded assets and liabilities were $800,000 and $180,000, respectively. Hart’s recorded assets and liabilities had fair values of $840,000 and $140,000, respectively. In Wall’s September 30, 20X1, balance sheet, what amount should be reported as goodwill?

$20,000

$160,000

$240,000

$180,000

A

$160,000

When, in the purchase of another company, the purchase price exceeds the fair value of all the assets the purchased company owns, then the excess is goodwill.

Purchase price $860,000
Fair value of assets $840,000
Less fair value of liabilities 140,000
Net fair value of assets 700,000
Goodwill $160,000
========

How well did you know this?
1
Not at all
2
3
4
5
Perfectly
55
Q

Neely Co. disclosed in the notes to its financial statements that a significant number of its unsecured trade account receivables are with companies that operate in the same industry. This disclosure is required to inform financial statement users of the existence of:

off-balance sheet risk of accounting loss.

concentration of credit risk.

risk of measurement uncertainty.

concentration of market risk.

A

concentration of credit risk.

Credit risk is the potential loss from any party to an agreement failing to perform. Credit risk must be disclosed.

Off-balance sheet risk occurs when the amount of a loss exceeds the related asset. Market risk disclosure is encouraged, but not required

How well did you know this?
1
Not at all
2
3
4
5
Perfectly
56
Q

Disclosure of such risk must be made if, based on management’s information, the following criteria are met:

a. the concentration exists at the ___of the financial statements,b. the concentration make the entity ___to the risk of a near-term severe impact, andc. it is at least ___possible that the events that could cause the severe impact will occur in the near term.

A

Date

vulnerable

reasonably

How well did you know this?
1
Not at all
2
3
4
5
Perfectly
57
Q

A ___impact is defined as a significant financially disruptive effect on the normal functioning of an entity.

A

severe

How well did you know this?
1
Not at all
2
3
4
5
Perfectly
58
Q

Examples of categories of concentrations include:

a. concentrations in the ___of business transacted with a particular customer, supplier, lender, grantor, or contributor,
b. concentrations in ___from particular products, services, or fund-raising events,
c. concentrations in the available sources of supply of materials, labor, or services or of licenses or other rights used in the entity’s operations, andd. concentrations in the market or ___area in which an entity conducts its operations.

A

Volume

Revenue

geographic area

How well did you know this?
1
Not at all
2
3
4
5
Perfectly
59
Q

What percentage is required for a concentration to be present?

A

10%

Examples of categories of concentrations include:

a. concentrations in the volume of business transacted with a particular customer, supplier, lender, grantor, or contributor,
b. concentrations in revenues from particular products, services, or fund-raising events,
c. concentrations in the available sources of supply of materials, labor, or services or of licenses or other rights used in the entity’s operations, andd. concentrations in the market or geographic area in which an entity conducts its operations.

How well did you know this?
1
Not at all
2
3
4
5
Perfectly
60
Q

TGR Enterprises provided the following information from its statement of financial position for the year ended December 31:

January 1__December 31
Cash $ 10,000 $ 50,000
Accounts receivable 120,000 100,000
Inventories 200,000 160,000
Prepaid expenses 20,000 10,000
Accounts payable 175,000 120,000
Accrued liabilities 25,000 30,000

TGR’s sales and cost of sales for the year were $1,400,000 and $840,000, respectively. What is the accounts receivable turnover, in days?

  1. 1
  2. 7
  3. 3
  4. 7
A

You will be given a key for all ratios. You dont really need to memorize this.

Accounting receivable turnover = Net credit sales ÷ Average receivables:

  • $1,400,000 ÷ (($120,000 + $100,000) ÷ 2) = 12.727 times in a year

Turnover in days = 365 days ÷ Turnover in a year:

  • 365 ÷ 12.727 = 28.7 (rounded)
How well did you know this?
1
Not at all
2
3
4
5
Perfectly
61
Q

A company reported the following information for Year 1:

Net income $34,000
Owner contribution 9,000
Deferred gain on an effective cash-
flow hedge 8,000
Foreign currency translation gain 2,000
Prior service cost not recognized in
net periodic pension cost 5,000

What is the amount of other comprehensive income for Year 1?

$14,000

$15,000

$43,000

$5,000

A

$5,000

Other comprehensive income includes items such as gains and losses on foreign currency transactions designated as hedges, gains and losses on derivative instruments, and gains or losses associated with pension or other postretirement benefits.

Therefore, for this question the correct answer is $5,000:

  • Deferred gain on an effective cash-flow hedge ($8,000) + Foreign currency translation gain ($2,000) − Prior service cost not recognized in net periodic pension cost ($5,000) = $5,000
How well did you know this?
1
Not at all
2
3
4
5
Perfectly
62
Q

According to the FASB conceptual framework, comprehensive income includes which of the following?

Neither loss on discontinued operations nor investment by owners

Investment by owners

Both loss on discontinued operations and investment by owners

Loss on discontinued operations

A

Loss on discontinued operations

SFAC 6 defines comprehensive income as: “Change in equity (net assets) of an entity during a period from transactions and other events and circumstances from nonowner sources. It includes all changes in equity during a period except those resulting from investments by owners and distributions to owners.”

Loss on discontinued operations is part of net income which changes equity; therefore, it is part of comprehensive income. By definition, investments by owners are specifically excluded from comprehensive income.

How well did you know this?
1
Not at all
2
3
4
5
Perfectly
63
Q

Which of the following statements about the refinancing of short-term obligations is incorrect?

The intent to refinance the short-term obligations is a required criterion to reclassify current liabilities as long term.

The ability to refinance the short-term obligations is a required criterion to reclassify current liabilities as long term.

Short-term obligations that are paid with cash before the balance sheet is released to the public can be refinanced and reclassified as long term.

The amount that can be refinanced is limited to the amount actually refinanced or the amount specified in a refinancing agreement even though an actual refinancing did not occur.

A

Short-term obligations that are paid with cash before the balance sheet is released to the public can be refinanced and reclassified as long term.

Short-term obligations intended to be refinanced cannot be reclassified as noncurrent if they are paid with current assets prior to the actual refinancing or a new refinancing agreement being agreed upon. The payment of cash would represent a settlement of the short-term obligation, not a refinancing.

All of the other answer choices are correct statements about the refinancing of short-term obligations to be reclassified as noncurrent.

How well did you know this?
1
Not at all
2
3
4
5
Perfectly
64
Q

Short-Term Obligations Expected to Be Refinanced

Short-term obligations arising from transactions in the normal course of business that are due in customary terms must be classified as current liabilities. Other short-term obligations may be excluded from current liabilities, but only if the enterprise:

a. intends to ___the obligation on a long-term basis andb. demonstrates the ability to ____the refinancing.

A

Refinance

Consummate (idk what this means - just memorize it)

How well did you know this?
1
Not at all
2
3
4
5
Perfectly
65
Q

Refinancing a short-term obligation on a long-term basis means:

a. replacing it with a ___-term obligation or equity securities orb. renewing, extending, or replacing it with short-term obligations for an uninterrupted period extending beyond ___ year(s) (or the operating cycle, if applicable) from the date of an enterprise’s balance sheet.

A

long

one year

How well did you know this?
1
Not at all
2
3
4
5
Perfectly
66
Q

The ability to consummate the refinancing may be demonstrated in either of the following two ways:

  1. By actual ___of a long-term obligation or equity security after the balance sheet date, but before the balance sheet is issued, for the purpose of refinancing the short-term obligation2. By entering into a financing ___, before the balance sheet is issued, that clearly permits the enterprise to refinance the short-term obligation on a long-term basis on terms that are readily determinable
A

issuance

agreement

How well did you know this?
1
Not at all
2
3
4
5
Perfectly
67
Q

Papillon Corp. sold goods to its 90%-owned subsidiary, Trook Corp, during 20X6. At the end of 20X6, 1/4th of these goods were included in Trook’s ending inventory. In its income statement for 20X6, Papillon reported freight-out expenses of $790,000, none of which was paid on sales made to Trook. Trook reported $375,000 of freight-out expenses for the same period. Trook’s freight-out expenses included $103,000 in freight costs paid to ship goods to Papillon. What amount of selling expenses should be reported in Papillon’s 20X6 consolidated income statement?

$1,165,000

$1,072,300

$1,113,500

$1,062,000

A

$1,062,000

Since freight-out costs between Trook and Papillon are paid by the seller (Trook), they are not included in the value of inventory by the buyer (Papillon). Also, since they were paid on an intercompany sale, these costs should be eliminated from Papillon’s consolidated income statement. Thus, consolidated selling expenses for 20X6 are as follows:

  • Papillon total + Trook’s total – Trook’s Intercompany
  • ($790,000) + ($375,000 – $103,000)
  • $790,000* + $272,000 = $1,062,000
How well did you know this?
1
Not at all
2
3
4
5
Perfectly
68
Q

Envoy Co. manufactures and sells household products. Envoy experienced losses associated with its small appliance group. Operations and cash flows for this group can be clearly distinguished from the rest of Envoy’s operations. In a strategic shift, Envoy plans to sell the small appliance group with its operations. What is the earliest point at which Envoy should report the small appliance group as a discontinued operation?

When Envoy classifies it as held for sale

When Envoy first sells any of the assets of the segment

When Envoy sells the majority of the assets of the segment

When Envoy receives an offer for the segment

A

When Envoy classifies it as held for sale

Discontinued operations are presented in a separate section of the income statement after income from continuing operations.

The discontinued operations section reflects the results of operations of an entity that is classified for sale or has actually been disposed of.

How well did you know this?
1
Not at all
2
3
4
5
Perfectly
69
Q

At the end of the accounting period, the components of other comprehensive income are transferred to which of the following stockholders’ equity accounts?

Retained earnings

Treasury stock

Additional paid-in capital (common stock)

Accumulated other comprehensive income

A

Accumulated other comprehensive income

The total of other comprehensive income for a period is transferred to a component of equity that is presented in the statement of financial position separately from retained earnings and additional paid-in capital.

This element of stockholders’ equity should carry an appropriate title, such as accumulated other comprehensive income.

The accumulated balances of each separate classification of that component of stockholders’ equity is required, either in the statement of financial position or in notes to the financial statements.

The classifications of other comprehensive income must be consistent throughout the financial statements.

How well did you know this?
1
Not at all
2
3
4
5
Perfectly
70
Q

Strut Co. has a payable to its parent, Plane Co. In which of the following balance sheets should this payable be reported separately?

Neither Strut’s balance sheet nor Plane’s consolidated balance sheet

Plane’s consolidated balance sheet

Both Strut’s balance sheet and Plane’s consolidated balance sheet

Strut’s balance sheet

A

Strut’s balance sheet

The payable to Plane Co. from Strut Co. would only appear on Strut Co.’s balance sheet. Payable and receivable amounts that are due to and due from subsidiaries of a consolidated business entity are not reported on the consolidated balance sheet of the consolidated company.

The payable would be eliminated during the consolidation process and would not appear on the balance sheet of the consolidated entity or the standalone balance sheet of the parent.

How well did you know this?
1
Not at all
2
3
4
5
Perfectly
71
Q

Which of the following would be reported as an investing activity in a company’s statement of cash flows?

Collection of proceeds from a note payable

Collection of a note receivable from a related party

Collection of an overdue account receivable from a customer

Collection of a tax refund from the government

A

Collection of a note receivable from a related party

Investing activities involve asset transactions other than those related to operating results (e.g., accounts receivables from sales and taxes).

How well did you know this?
1
Not at all
2
3
4
5
Perfectly
72
Q

Which of the following statements is correct concerning the appearance of noncontrolling interest on the income statement?

None of the answer choices are appropriate disclosure of the noncontrolling interest on the income statement.

Revenues, expenses, gains, losses, net income or loss, and other comprehensive income are reported in the consolidated financial statements as the amounts attributable to the owners of the parent, followed by a separate disclosure of the revenues, expenses, gains, losses, net income or loss, and other comprehensive income attributable to the noncontrolling interest.

Revenues, expenses, gains, losses, net income or loss, and other comprehensive income are reported in the consolidated financial statements at the owners’ amounts with disclosure of the noncontrolling interest only in the footnotes.

Revenues, expenses, gains, losses, net income or loss, and other comprehensive income are reported in the consolidated financial statements at the consolidated amounts, which include the amounts attributable to the owners of the parent and the noncontrolling interest.

A

Revenues, expenses, gains, losses, net income or loss, and other comprehensive income are reported in the consolidated financial statements at the consolidated amounts, which include the amounts attributable to the owners of the parent and the noncontrolling interest.

FASB ASC 810-10-45-19 requires that the consolidated amounts of these items (revenues, expenses, gains, losses, net income or loss, and other comprehensive income) be reported on the income statement.

The amount of consolidated net income attributable to the parent and to the noncontrolling interest must be clearly identified and presented on the face of the consolidated statement of income.

How well did you know this?
1
Not at all
2
3
4
5
Perfectly
73
Q

A holder of a variable interest that is not the primary beneficiary acquired additional variable interests in the variable interest entity (VIE). What action, if any, should follow?

No action is necessary because the primary beneficiary of a VIE does not change subsequent to the initial assessment.

The holder of the variable interest should use the voting-interest model to determine whether the VIE should be consolidated.

The primary beneficiary should discontinue consolidation of the VIE because the election to consolidate is no longer allowed.

The holder of the variable interest should reconsider whether it is now the primary beneficiary.

A

The holder of the variable interest should reconsider whether it is now the primary beneficiary.

When the holder of a variable interest that is not the primary beneficiary increases the level of interest the holder has in the variable interest entity, it must assess if the increased interest has now made the holder the primary beneficiary and therefore possibly subject to consolidation procedures. The primary beneficiary is subject to change as facts change, making a reassessment necessary.

Without evidence to the contrary, the primary beneficiary should continue to consolidate the entity. No information is given that would cause the primary beneficiary to believe they are not the primary beneficiary. We are given no information about voting or ownership percentages, making the voting-interest model not applicable.

How well did you know this?
1
Not at all
2
3
4
5
Perfectly
74
Q

An entity is known as a variable interest entity and is subject to consolidation if, by design, either of the following conditions (a. or b.) exists:

a. The total equity investment at risk is not sufficient to permit the entity to finance its activities without additional subordinated financial support from other parties.
b. As a group the holders of the equity investment at risk lack any one or more of the following three characteristics of a controlling financial interest:
(1) The power, through voting rights or similar rights, to direct the activities of an entity that most significantly impact the entity’s economic performance
(2) The obligation to absorb the expected losses of the entity if they occur
(3) The right to receive the expected residual returns of the entity if they occur

A

Yep

How well did you know this?
1
Not at all
2
3
4
5
Perfectly
75
Q

An entity is known as a variable interest entity and is subject to consolidation if, by design, either of the following conditions (a. or b.) exists:

a. The total equity investment at risk is not sufficient to permit the entity to finance its activities without additional subordinated financial ___from other parties.
b. As a group the holders of the equity investment at risk lack any one or more of the following three characteristics of a controlling financial interest:
(1) The power, through ___rights or similar rights, to direct the activities of an entity that most significantly impact the entity’s economic performance
(2) The obligation to absorb the expected ___of the entity if they occur
(3) The right to receive the expected residual ___of the entity if they occur

A

support

voting

losses

returns

How well did you know this?
1
Not at all
2
3
4
5
Perfectly
76
Q

Where in its financial statements should a company disclose information about its concentration of credit risks?

No disclosure is required.

Management’s report to shareholders

The notes to the financial statements

Supplementary information to the financial statements

A

FASB ASC 825-10-50-20 requires note disclosures regarding a company’s concentrations of credit risks.

How well did you know this?
1
Not at all
2
3
4
5
Perfectly
77
Q

Whitt Co. prepares its statement of cash flows using the indirect method. Whitt’s unamortized bond premium account decreased by $18,500 during the year. How should Whitt report the change in unamortized bond premium in its statement of cash flows?

As a subtraction from net income in the operating activities section

As a financing cash outflow

As a financing cash inflow

As an addition to net income in the operating activities section

A

As a subtraction from net income in the operating activities section

The amortization of a bond premium is the difference between cash interest and interest expense. Cash paid for interest is reported in operating activities. Under the indirect method, interest expense is already included in net income. Amortization of a premium on bonds payable results in the interest expense amount being less than cash paid.

Because less expense has been deducted in computing income than the amount of cash paid for interest, the difference (captured in the change in the bond discount account) must be subtracted from income to reconcile to the cash provided or used for operating activities.

How well did you know this?
1
Not at all
2
3
4
5
Perfectly
78
Q

Which of the following should be disclosed in a summary of significant accounting policies?

Future lease payments in the aggregate and for each of the five succeeding fiscal years

Depreciation expense

Composition of sales by segment

Basis of profit recognition on long-term construction contracts

A

Basis of profit recognition on long-term construction contracts

Only basis of profit recognition on long-term construction contracts is disclosure related to an accounting method.

The other answer choices might be included in disclosure related to those specific expenses, but are not accounting methods that would be included in the significant accounting policies footnote.

How well did you know this?
1
Not at all
2
3
4
5
Perfectly
79
Q

Contract balances: An entity should disclose all of the following:

  1. The opening and closing ___of receivables, contract assets, and contract liabilities from contracts with customers, if not otherwise separately presented or disclosed
  2. Revenue ____ in the reporting period that was included in the contract liability balance at the beginning of the period
  3. Revenue recognized in the reporting period from ____ ____ satisfied or partially satisfied in previous periods; for example, changes in transaction price
A

balances

recognized

performance obligations

How well did you know this?
1
Not at all
2
3
4
5
Perfectly
80
Q

FASB ASC 235-10-05-3 requires disclosure of significant___ policies.

The accounting policies of an entity are the specific accounting principles and the methods of applying those principles that are judged by the management of the entity to be the most appropriate in the circumstances to present fair ___ ___

A

accounting

financial reporting

How well did you know this?
1
Not at all
2
3
4
5
Perfectly
81
Q

Baler Co. prepared its statement of cash flows at year-end using the direct method. The following amounts were used in the computation of cash flows from operating activities:

Beginning inventory $ 200,000
Ending inventory 150,000
Cost of goods sold 1,200,000
Beginning accounts payable 300,000
Ending accounts payable 200,000

What amount should Baler report as cash paid to suppliers for inventory purchases?

$1,350,000

$1,200,000

$1,250,000

$1,300,000

A

$1,250,000

How well did you know this?
1
Not at all
2
3
4
5
Perfectly
82
Q

Yellow Co. received a large worker’s compensation claim of $90,000 in the third quarter for an injury occurring in the third quarter. How should Yellow account for the transaction in its interim financial report?

Recognize $90,000 in the third quarter

Recognize $30,000 for each of the first three quarters

Recognize $22,500 ratably over the four quarters of the year

Disclose the $90,000 in the third quarter and recognize it at year-end

A

Recognize $90,000 in the third quarter

In general, interim financial reports should be based on the principles, practices, and policies used in the preparation of the last annual report.

Deferrals, accruals, and estimations at the end of each interim period are determined on the same basis as the same judgments would be made for an annual period; hence Yellow would recognize the entire $90,000 in the third quarter.

How well did you know this?
1
Not at all
2
3
4
5
Perfectly
83
Q

During the year, Verity Co. purchased $200,000 of Otra Co. bonds at par and $50,000 of U.S. Treasury bills. Verity classified the Otra bonds as available-for-sale securities and the Treasury bills as cash equivalents. In Verity’s statement of cash flows, what amount should it report as net cash used in investing activities?

$0

$200,000

$250,000

$150,000

A

$200,000

Cash payments to acquire debt instruments of other entities are classified as cash outflows from investing activities on the statement of cash flows. The purchase of Otra Co. bonds would represent a net cash used of $200,000.

Cash paid for items classified as cash and cash equivalents are not reported on the statement of cash flows because they are merely a transfer of one type of cash for another type of cash.

How well did you know this?
1
Not at all
2
3
4
5
Perfectly
84
Q

Cash equivalents are short-term, highly liquid investments that:

  1. are readily ___to known amounts of cash and
  2. are so near maturity that they represent insignificant risk of changes in value due to changes in interest rates. (Generally, only investments with original maturities of ___ months or less qualify as cash equivalents, such as Treasury bills, commercial paper, money market funds, and federal funds sold.)
A

convertible

three months

How well did you know this?
1
Not at all
2
3
4
5
Perfectly
85
Q

Treasury bills, commercial paper, money market funds, and federal funds sold.

These are examples of what?

A

Cash equivalents

How well did you know this?
1
Not at all
2
3
4
5
Perfectly
86
Q

During 20X2, Solomon Co. purchased equipment for cash of $128,000, and sold equipment with a $38,000 carrying value for a loss of $14,000. How should these transactions be reported in Solomon’s 20X2 statement of cash flows?

Cash inflow of $24,000 and cash outflow of $128,000

Cash outflow of $142,000

Cash outflow of $104,000

Cash outflow of $152,000

A

Cash inflow of $24,000 and cash outflow of $128,000

According to FASB ASC 230-10-45-13, cash flows associated with transactions involving long-term assets are classified as cash flows from investing activities.

Cash inflows are not to be netted against cash outflows.

There is a cash outflow of $128,000 to purchase equipment given directly in the problem. The cash inflow must be determined from the facts as given. A loss on disposal indicates that the cash received is lower than the carrying value.

Carrying value $38,000
Loss on disposal (14,000)
Cash selling price $24,000
=======

How well did you know this?
1
Not at all
2
3
4
5
Perfectly
87
Q

On January 2 of the current year, Peace Co. paid $310,000 to purchase 75% of the voting shares of Surge Co. Peace reported retained earnings of $80,000, and Surge reported contributed capital of $300,000 and retained earnings of $100,000.

The purchase differential was attributed to depreciable assets with a remaining useful life of 10 years. Peace used the equity method in accounting for its investment in Surge.

Surge reported net income of $20,000 and paid dividends of $8,000 during the current year. Peace reported income, exclusive of its income from Surge, of $30,000 and paid dividends of $15,000 during the current year.

What amount will Peace report as dividends declared and paid in its current year’s consolidated statement of retained earnings?

$23,000

$21,000

$8,000

$15,000

A

$15,000

Only dividends paid to Peace shareholders will be reported as dividends paid. Dividends paid to Peace by Surge will be eliminated in consolidation.

Dividends paid to shareholders other than Peace will be reported as an adjustment to the noncontrolling interest account.

How well did you know this?
1
Not at all
2
3
4
5
Perfectly
88
Q

Green Co. had the following equity transactions at December 31:

Cash proceeds from sale of investment in Blue Co.
(carrying value $60,000) $75,000
Dividends received on Grey Co. stock 10,500
Common stock purchased from Brown Co. 38,000

What amount should Green recognize as net cash from investing activities in its statement of cash flows at December 31?

$47,500

$75,000

$37,000

$85,500

A

$37,000

Cash proceeds from the sale of an investment are a cash inflow and cash paid to purchase stock is a cash outflow. Both are investing activities.

  • $75,000 - $38,000 = $37,000
How well did you know this?
1
Not at all
2
3
4
5
Perfectly
89
Q

Karr, Inc., reported net income of $300,000 for 20X1. Changes occurred in several balance sheet accounts as follows:

Equipment $25,000 increase
Accumulated depreciation 40,000 increase
Note payable 30,000 increase

Additional Information

  • During 20X1, Karr sold equipment costing $25,000, with accumulated depreciation of $12,000, for a gain of $5,000.
  • In December 20X1, Karr purchased equipment costing $50,000 with $20,000 cash and a 12% note payable of $30,000.
  • Depreciation expense for the year was $52,000.

In Karr’s 20X1 statement of cash flows, net cash used in investing activities should be:

$35,000.

$12,000.

$2,000.

$22,000.

A
How well did you know this?
1
Not at all
2
3
4
5
Perfectly
90
Q

A company is preparing its year-end cash flow statement using the indirect method. During the year, the following transactions occurred:

Dividends paid $300
Proceeds from the issuance of common stock 250
Borrowings under a line of credit 200
Proceeds from the issuance of convertible bonds 100
Proceeds from the sale of a building 150

What is the company’s increase in cash flows provided by financing activities for the year?

$250

$50

$350

$150

A

$250

The company’s increase in cash flows is $250, calculated as follows:

Dividends paid $(300)
Proceeds from the issuance of common stock 250
Borrowings under a line of credit 200
Proceeds from the issuance of convertible bond 100
Net increase from financing activities $250

The proceeds from the sale of a building are included in investing activities.

How well did you know this?
1
Not at all
2
3
4
5
Perfectly
91
Q

Which of the following statements about reporting discontinued operations in the balance sheet is incorrect?

The assets and liabilities of the component of the entity can be reported as a single or net amount.

If the discontinued operation includes both assets and liabilities, they must be presented separately in the respective asset and liability sections of the entity’s balance sheet.

The discontinued operation must be reported separately in the balance sheet in the period in which it is classified as held for sale.

If prior periods are presented, the assets and liabilities classified as held for sale must be separated out in prior periods as well as in the period in which the assets are initially classified as held for sale.

A

The assets and liabilities of the component of the entity can be reported as a single or net amount.

The assets and liabilities of a component that qualify as discontinued operations need to be reported separately from assets in use on the balance sheet.

When such reporting occurs, the assets need to be separated in a similar manner in any prior periods presented.

When a component has both assets and liabilities, they are not to be “netted,” but rather the assets should be reported in the asset section of the balance sheet and the liabilities should be reported in the liability section of the balance sheet.

*

How well did you know this?
1
Not at all
2
3
4
5
Perfectly
92
Q

A company had the following transactions during the year:

Principal payments on notes payable $48,000
Interest payments on notes payable 8,000
Cash payment to purchase 100 shares of
another company’s common stock 25,000

What amount is classified as cash outflow for financing activities in the company’s statement of cash flows?

$48,000

$81,000

$56,000

$73,000

A

$48,000

Financing activities are those activities that provide financing or use financing. They include the issuance and repurchase of a company’s own stock, cash dividend payments to shareholders, and the issuance and repurchase of debt. Interest on debt is an operating activity while purchase of another company’s stock is an investing activity.

The only answer choice that meets the definition of financing is principal payments on notes payable for $48,000, which is a form of debt repayment.

How well did you know this?
1
Not at all
2
3
4
5
Perfectly
93
Q

Sanni Co. had $150,000 in cash-basis pretax income for the year. At the current year-end, accounts receivable decreased by $20,000 and accounts payable increased by $16,000 from their previous year-end balances. Compared to the accrual-basis method of accounting, Sanni’s cash-basis pretax income is:

higher by $4,000.

higher by $36,000.

lower by $36,000.

lower by $4,000.

A

higher by $36,000.

Relative to accrual basis, a decrease in accounts receivable is an increase in cash because cash must be received to decrease accounts receivable.

Relative to accrual basis, an increase in accounts payable is an increase in cash because accounts payable was increased instead of making cash purchases.

Decrease in accounts receivable $20,000
Increase in accounts payable 16,000
Total increase in cash-basis income $36,000

How well did you know this?
1
Not at all
2
3
4
5
Perfectly
94
Q

Financial statement line item explanations include which of the following?

Degree of credit or nonperformance risk

Segment reporting

Potential litigation

Inability to maintain a qualified workforce

A

Degree of credit or nonperformance risk

Many financial statement line item explanations, such as local denomination demand deposits, do not require further explanatory information. However, other line items require varying degrees of disclosure. A summary of potential additional disclosures is as follows:

  • For assets: the nature, quality, and location; future cash flows; relation to other line items; and significant contractual, statutory, regulatory, or judicial restrictions.
  • For assets and liabilities resulting from financial instruments or other contracts: contractual or legal terms (e.g., timing of receipts and disbursements), degree of credit or nonperformance risk, potential effect related to inability to pay or perform, and method used to determine the cash flows.
  • Other disclosures could include equity instrument terms or conditions, potential effects of changing accounting methods, breakdown of aggregated line items, alternative measurements, and the line item’s relation to other line items.
How well did you know this?
1
Not at all
2
3
4
5
Perfectly
95
Q

When entities face risk due to a lack of diversification, they must include disclosures in the notes to the financial statements about “vulnerability to concentrations.” Which of the following is not an example of a concentration to which an entity may be considered vulnerable?

An entity relies on certain available sources of raw materials, labor, services, or licenses.

An entity manufactures products that are used in a wide variety of industries.

An entity operates primarily in the specific market or geographic region in which an entity conducts its operations.

An entity has a large volume of business with one customer, supplier, lender, or contributor.

A

An entity manufactures products that are used in a wide variety of industries.

Categories of concentrations include a large volume of business with one customer, supplier, lender, contributor, or grantor. Categories also include when an entity operates primarily in the specific market or geographic region in which an entity conducts its operations and when an entity relies heavily on certain available sources of raw materials, labor, services, licenses, or other rights.

Finally, a category exists when an entity has a concentration in revenue form particular products, services, or fundraising events.

An entity that manufactures products that are used in a wide variety of industries would have a lower, not higher, vulnerability to concentrations.

How well did you know this?
1
Not at all
2
3
4
5
Perfectly
96
Q

Mirr, Inc., was incorporated on January 1, 20X0, with proceeds from the issuance of $750,000 in stock and borrowed funds of $110,000. During the first year of operations, revenues from sales and consulting amounted to $82,000, and operating costs and expenses totaled $64,000. On December 15, 20X0, Mirr declared a $3,000 cash dividend, payable to stockholders on January 15, 20X1. No additional activities affected owners’ equity in 20X0. Mirr’s liabilities increased to $120,000 by December 31, 20X0. On Mirr’s December 31, 20X0, balance sheet (statement of financial position), total assets should be reported at:

$875,000.

$882,000.

$885,000.

$878,000.

A
How well did you know this?
1
Not at all
2
3
4
5
Perfectly
97
Q

___: Probable future economic benefits obtained or controlled by a particular entity as a result of past transactions or events

___: probable future sacrifices of economic benefits arising from present obligations to transfer assets or provide services to other entities as a result of past transactions

___: The residual interest in the assets of an entity that remains after deducting its liabilities. For a corporation, equity is the ownership interest

A

Assets

Liabilities

Equtiy

How well did you know this?
1
Not at all
2
3
4
5
Perfectly
98
Q

A general principle of accounting is that the offsetting of assets and liabilities in the balance sheet is improper except where a right of ___exists.

A

setoff

How well did you know this?
1
Not at all
2
3
4
5
Perfectly
99
Q

A debtor with a payable to an entity MAY offset a receivable from that same entity and display only the difference as a net payable or receivable T/F

A

False – they MAY NOT

How well did you know this?
1
Not at all
2
3
4
5
Perfectly
100
Q

For example, a debtor with a payable to an entity may not offset a receivable from that same entity and display only the difference as a net payable or receivable unless the following specified conditions are met for the right of setoff to exist:

a. Each of the two parties owes the other ___amounts.
b. The reporting party has the right to set off the amount owed with the amount owed by the other party. T/F
c. The reporting entity ___ to set off.
d. The right of setoff is enforceable at ___

A

determinable

True

Intends

law

How well did you know this?
1
Not at all
2
3
4
5
Perfectly
101
Q

A public entity sells steel for use in construction. One of its customers accounts for 43% of sales, and another customer accounts for 40% of sales. What should the entity disclose in its annual financial statements about these two customers?

The financial condition of the two customers

The amount of the entity’s revenue from each of the two customers

The names of the two customers

The payment terms of accounts receivable due from each of the two customers

A

The amount of the entity’s revenue from each of the two customers

If revenues from transactions with a single external customer amount to 10% or more of an entity’s revenues, the business must disclose that fact, the total amount of revenues from each such customer, and the identity of the segments reporting the revenues.

A major customer is a single customer, or a group of entities, known to a reporting enterprise to be under common control.

The identity, payment terms, and financial condition of the customers do not need to be disclosed.

How well did you know this?
1
Not at all
2
3
4
5
Perfectly
102
Q

If revenues from transactions with a single external customer amount to __% or more of an enterprise’s revenues, the enterprise must disclose that fact, the total amount of revenues from each such customer, and the identity of the segments reporting the revenues.

A

10%

How well did you know this?
1
Not at all
2
3
4
5
Perfectly
103
Q

Statement of Financial Accounting Concepts 8 (SFAC 8), Chapter 8, lists four limitations/constraints to consider related to disclosure requirements. They are:

representational faithfulness, materiality, future-oriented information, and predictive value.

cost constraint, materiality, potential adverse consequences, and historical cost.

cost constraint, potential adverse consequences, future-oriented information, and relevance.

relevance, representational faithfulness, materiality, and predictive value.

A

cost constraint, potential adverse consequences, future-oriented information, and relevance.

  1. Relevance: Disclosure is based upon relevance, not entity-specific materiality.
  2. Cost constraint: The FASB has an expectation that financial statement users have awareness of accounting rules, policies, and regulations; thus, common knowledge can be excluded from the notes. Disclosure should include details of measurement if alternatives exist, methods not obvious to the user, or methods if changed since prior reporting.
  3. Potential adverse consequences: The FASB will consider potential adverse consequences. Disclosure can have both beneficial and adverse consequences.
  4. Future-oriented information: The FASB does not require entities to disclose predictions of future outcomes that could result in negative consequences. However, two types of forward-looking information are useful and should be provided: (1) estimates and assumptions, and (2) management’s existing plans and strategies for management-controlled matters.
How well did you know this?
1
Not at all
2
3
4
5
Perfectly
104
Q
  1. ___: Disclosure is based upon relevance, not entity-specific materiality.
  2. ___ ___: The FASB has an expectation that financial statement users have awareness of accounting rules, policies, and regulations; thus, common knowledge can be excluded from the notes. Disclosure should include details of measurement if alternatives exist, methods not obvious to the user, or methods if changed since prior reporting.
  3. ___ ___ ___: The FASB will consider potential adverse consequences. Disclosure can have both beneficial and adverse consequences.
  4. ___-___ ___: The FASB does not require entities to disclose predictions of future outcomes that could result in negative consequences. However, two types of forward-looking information are useful and should be provided: (1) estimates and assumptions, and (2) management’s existing plans and strategies for management-controlled matters.
A

Relevance

Cost Constraint

Potential Adverse Consequences

Future-oriented information

How well did you know this?
1
Not at all
2
3
4
5
Perfectly
105
Q

For purposes of consolidating financial interests, a majority voting interest is deemed to be:

greater than 50% of the directly or indirectly owned outstanding voting shares of another entity.

50% of the directly or indirectly owned outstanding voting shares of another entity.

50% of the directly or indirectly owned outstanding voting shares and at least 50% of the directly or indirectly owned outstanding nonvoting shares of another entity.

greater than 50% of the directly or indirectly owned outstanding voting shares and at least 50% of the directly or indirectly owned outstanding nonvoting shares of another entity.

A

greater than 50% of the directly or indirectly owned outstanding voting shares of another entity.

GAAP requires that consolidated financial statements be prepared when one of the entities in the group directly or indirectly has a controlling financial interest in the other entities.

FASB ASC 810 specifies that, in general, the usual condition for consolidated financial statements is ownership (direct or indirect) of a majority voting interest (i.e., at least one share in excess of 50%).

How well did you know this?
1
Not at all
2
3
4
5
Perfectly
106
Q

During 20X1, Beck Co. purchased equipment for cash of $47,000, and sold equipment with a $10,000 carrying value for a gain of $5,000. How should these transactions be reported in Beck’s 20X1 statement of cash flows?

Cash inflow of $5,000 and cash outflow of $47,000

Cash inflow of $15,000 and cash outflow of $47,000

Cash outflow of $32,000

Cash outflow of $42,000

A

Cash inflow of $15,000 and cash outflow of $47,000

How well did you know this?
1
Not at all
2
3
4
5
Perfectly
107
Q

In its 20X1 income statement, Kilm Co. reported cost of goods sold of $450,000. Changes occurred in several balance sheet accounts as follows:

Inventory $160,000 decrease
Accounts payable—suppliers 40,000 decrease

What amount should Kilm report as cash paid to suppliers in its 20X1 cash flow statement, prepared under the direct method?

$570,000

$250,000

$650,000

$330,000

A

$330,000

How well did you know this?
1
Not at all
2
3
4
5
Perfectly
108
Q

A company has the following liabilities at year-end:

Mortgage note payable; $16,000 due within 12 months $355,000
Short-term debt that the company is refinancing
with long-term debt 175,000
Deferred tax liability arising from depreciation 25,000

What amount should the company include in the current liability section of the balance sheet (statement of financial position)?

$41,000

$191,000

$0

$16,000

A

$16,000

Only the current portion of the mortgage is included in current liabilities.

All deferred tax liabilities and deferred tax assets are classified as noncurrent. The refinanced loan is not included in current liabilities. The FASB states that a short-term obligation should be excluded from current liabilities if the entity intends to refinance the obligation on a long-term basis, and that intent is supported by an ability to consummate the refinancing.

How well did you know this?
1
Not at all
2
3
4
5
Perfectly
109
Q

Current liabilities represent obligations whose liquidation is expected to require the use of current assets or the creation of other current liabilities, and include the following:

a. Obligations for items that have entered into the ___ ___
b. Collections received in ___of the delivery of goods or performance of services
c. Debts arising from operations directly related to the operating cycle (e.g., accruals for wages, salaries, commissions, rentals, royalties, and income and other taxes) T/F
d. Other liabilities whose regular and ordinary liquidation is expected to occur within ___ ___ or less (e.g., dividends payable, warranty payable, interest payable)

A

Operating Cycle (operating cycle is within 1 year)

advance (unearned revenue)

True

one year

How well did you know this?
1
Not at all
2
3
4
5
Perfectly
110
Q

On December 31, 20X1, Date Co. awaits judgment on a lawsuit for a competitor’s infringement of Date’s patent. Legal counsel believes it is probable that Date will win the suit and indicated the most likely award together with a range of possible awards. How should the lawsuit be reported in Date’s 20X1 financial statements?

By accrual for the lowest amount of the range of possible awards

In note disclosure only

Neither in note disclosure nor by accrual

By accrual for the most likely award

A

In note disclosure only

If Date Co. wins the lawsuit, the award paid to Date will be a gain.

FASB ASC 450-30-50-1 provides that gain contingencies should not be reflected in the accounts (i.e., accrued) but that adequate disclosure should be made in notes to the financial statements.

A loss contingency would be reported by accrual for the most likely award or for the lowest amount of the range of possible awards if no amount can be considered most likely. (FASB ASC 450-20-25-4)

How well did you know this?
1
Not at all
2
3
4
5
Perfectly
111
Q

An important element of the topic of risks and uncertainties is selectivity. Selectivity involves the specified criteria that serve to screen the ___and ___encountered by every entity

The types of risks and uncertainties discussed in this section are:

a. the nature of the entity’s ___,
b. the use of ___in the preparation of the entity’s financial statements, and
c. significant ___in certain aspects of the entity’s operations.

A

risks and uncertainties

a. Operations
b. estimates
c. concentrations

How well did you know this?
1
Not at all
2
3
4
5
Perfectly
112
Q

Which of the following information should be included in Gold Corporation’s 20X7 summary of significant accounting policies?

The specific amounts of the components of pension expense

The specific amounts of raw material inventory, work-in-process inventory, and finished goods shown in aggregate on the balance sheet
The policies regarding inventory valuation and the methods used for inventory cost determination

The valuation model used to determine the value of stock options granted to upper-level managers

A

The policies regarding inventory valuation and the methods used for inventory cost determination

FASB ASC 235-10-50-4 requires a description of all significant accounting policies when financial statements are issued. A listing of required policy disclosures by this pronouncement includes basis of consolidation, depreciation methods, amortization of intangibles, inventory pricing, and several other items.

Gold should include information concerning how inventory is valued and the inventory cost flow assumptions used in its summary of significant accounting policies.

Note: While the other three information items in the answer choices should be disclosed in the notes to the financial statements, they should not be included in the summary of significant accounting policies.

How well did you know this?
1
Not at all
2
3
4
5
Perfectly
113
Q

Examples of accounting principles and methods for which disclosure of policy is frequently made include, but are not limited to, the following:

a. Depreciation methods
b. Consolidation basis
c. Interperiod tax allocation
d. Inventory pricing
e. Revenue recognition methods

A

Yep!

How well did you know this?
1
Not at all
2
3
4
5
Perfectly
114
Q

General purpose external financial reporting of a corporation focuses primarily on the needs of which of the following users?

Investors and creditors and their advisors

The board of directors of the corporation

Regulatory and taxing authorities

The management of the corporation

A

Investors and creditors and their advisors

General purpose financial statements are designed to provide information to the primary users since information cannot be provided directly to these users.

The primary users are existing and potential investors, lenders, and other creditors. These users make decisions about buying, selling, or holding equity and debt instruments or providing credit by evaluating the expected returns from their investment. These parties need information about the prospects of future net cash inflows to the entity.

They also need information about the entity’s resources, claims against those resources, and how efficiently the entity’s management and governing board have used the entity’s resources.

Regulatory and taxing authorities, the board of directors, and management of the corporation are also users of general purpose financial statements; however, they are not the primary users.

How well did you know this?
1
Not at all
2
3
4
5
Perfectly
115
Q

General purpose financial statements are designed to provide information to the ___users since information cannot be provided directly to these users.

The primary users are ___and ___investors, lenders, and other creditors. These users make decisions about buying, selling, or providing credit by evaluating the expected returns from their investment. These parties need information about the prospects of future net cash inflows to the entity.

They also need information about the entity’s resources, claims against those resources, and how efficiently the entity’s management and governing board have used the entity’s resources. T/F

Regulatory and taxing authorities, the board of directors, and management of the corporation are also users of general purpose financial statements. These are primary users also. T/F

A

primary

existing and potential

True

False. Regulatory and taxing authorities, the board of directors, and management of the corporation are NOT primary users. They’re secondary users.

How well did you know this?
1
Not at all
2
3
4
5
Perfectly
116
Q

Which of the following does not represent an element of other comprehensive income under current generally accepted accounting principles?

Amortization of unrecognized prior service costs related to a pension plan

Cumulative effect of a change in accounting estimate

Accumulated gains and losses on available-for-sale debt investments

Foreign currency adjustments

A

Cumulative effect of a change in accounting estimate

The components of other comprehensive income are to be presented based on their nature. Under current authoritative accounting literature, three categories of elements of other comprehensive income exist:

Unrealized gains and losses on available-for-sale debt investments
Foreign currency items
Changes in unrecognized prior service costs, unrecognized gains and losses, and unrecognized transition assets or obligations related to defined benefit pension plans and defined benefit other postretirement plans

How well did you know this?
1
Not at all
2
3
4
5
Perfectly
117
Q

The components of other comprehensive income are to be presented based on their nature. Under current authoritative accounting literature, three categories of elements of other comprehensive income exist:

Unrealized gains and losses on ___-___-__ debt investments
Foreign _____items
Changes in unrecognized prior ___ ___, unrecognized gains and losses, and unrecognized transition assets or obligations related to defined benefit pension plans and defined benefit other postretirement plans

A

available for sale

Currency

service costs

How well did you know this?
1
Not at all
2
3
4
5
Perfectly
118
Q

Which of the following is included on a statement of changes in equity?

Events changing stockholders’ equity accounts are listed chronologically to the left.

All of the items listed are included on a statement of changes in equity.

The impact of the transactions on the number of shares of stock, if any, is presented in the descriptions to the left.

Column headings identify individual stockholders’ equity accounts.

A

All of the items listed are included on a statement of changes in equity.

A statement of changes in stockholders’ equity includes the following:

  • Column headings that identify individual stockholders’ equity accounts
  • Events changing stockholders’ equity accounts
  • The body of the statement presented in terms of the dollar impact of various transactions and events
  • The impact of the transactions on the number of shares of stock, if any
  • Ending balances that tie to the items presented in the stockholders’ equity section of the balance sheet on the same dates
How well did you know this?
1
Not at all
2
3
4
5
Perfectly
119
Q

A statement of changes in stockholders’ equity includes the following:

  • ___headings that identify individual stockholders’ equity accounts
  • ___changing stockholders’ equity accounts
  • The body of the statement presented in terms of the dollar impact of various transactions and events T/F
  • The impact of the transactions on the number of shares of ___, if any
  • ___balances that tie to the items presented in the stockholders’ equity section of the balance sheet on the same dates
A

Column

Events

True

Stock

ending

How well did you know this?
1
Not at all
2
3
4
5
Perfectly
120
Q

Consolidated financial statements are typically prepared when one company has a controlling financial interest in another, unless:

the fiscal year-ends of the two companies are more than three months apart.

the subsidiary is a finance company.

such control does not rest with the majority owner because the subsidiary is in bankruptcy.

the two companies are in unrelated industries, such as manufacturing and real estate.

A

such control does not rest with the majority owner because the subsidiary is in bankruptcy.

Prior to FASB ASC 810-10-15, some parent companies would defend their decision to not consolidate certain subsidiaries by arguing that the nature of the subsidiary’s business was too unlike that of the parent.

For example, the subsidiary might be a financial institution and the parent a manufacturing operation.

FASB ASC 810-10-15 eliminated such “non-homogenous operations” defenses for not consolidating majority-owned subsidiaries by specifying that all majority-owned subsidiaries have to be consolidated unless control does not rest with the majority owner.

121
Q

Statement of Financial Accounting Concepts 8 (SFAC 8), Chapter 8, lists all of the following examples of past events and current conditions potentially impacting the entity’s future line items and cash flows except for:

suspected regulatory violations.

related party reporting.

existing or potential litigation.

dependency on a few customers or suppliers.

A

related party reporting.

Examples of past events and current conditions potentially impacting the entity’s future line items and cash flows but which have not yet been incorporated into financial statement line items include existing or potential litigation; suspected or known statute, judicial, regulatory, or contract violations; unrecognized existing commitments expected to be recognized in the future; events where significant uncertainty led to the decision to not recognize the event; and subsequent events

. Items not necessarily impacting line items that may require disclosure include dependency on one or a few customers or suppliers for profitability, input or output market volatility, uncertainty regarding an entity’s access to markets for inputs or outputs or ability to maintain a qualified workforce, and other significant specific entity risk.

Related party reporting is considered to be a reporting entity disclosure.

122
Q

During the current year, Cooley Co. had an unrealized gain of $100,000 on a debt investment classified as available-for-sale. Cooley’s corporate tax rate is 25%. What amount of the gain should be included in Cooley’s net income and other comprehensive income at the end of the current year?

Net income $25,000
Other comprehensive income $75,000

Net income $75,000
Other comprehensive income $25,000

Net income $0
Other comprehensive income $75,000

Net income $100,000
Other comprehensive income $0

A

Net income $0
Other comprehensive income $75,000

Available-for-sale debt securities are carried on the balance sheet at fair value. Unrealized changes in fair value between periods are reported in other comprehensive income for the period. Gains and losses are not reported on the income statement until realized. Items in other comprehensive income are reported net of their effective tax.

The unrealized gain in other comprehensive income is $75,000 ($100,000 gain × (1 – .25 tax rate)).

123
Q

Available-for-sale (AFS) securities are carried on the balance sheet at ___ value.

A

fair

124
Q

Available-for-sale securities

Unrealized gains and losses from changes in fair value are reported in ___

Unrealized gains or losses from sold securities are adjusted when the entire portfolio is evaluated for fair value at date of sale. T/F

A

Other Comprehensive Income (OCI)
False - it is evaluated at YEAR-END

125
Q

Wagner, a holder of a $1,000,000 Palmer, Inc., bond, collected the interest due on March 31, 20X1, and then sold the bond to Seal, Inc., for $975,000. On that date, Palmer, a 75% owner of Seal, had a $1,075,000 carrying amount for this bond. What was the effect of Seal’s purchase of Palmer’s bond on the retained earnings and noncontrolling (minority) interest amounts reported in Palmer’s March 31, 20X1, consolidated balance sheet?

Retained earnings: $75,000 increase; Noncontrolling interest: $25,000 increase

Retained earnings: $0; Noncontrolling interest: $25,000 increas

Retained earnings: $100,000 increase; Noncontrolling interest: $0

Retained earnings: $0; Noncontrolling interest: $100,000 increase

A

Retained earnings: $100,000 increase; Noncontrolling interest: $0

Carrying value of Palmer bonds payable $1,075,000
Less acquisition cost to Seal, Inc. 975,000
Gain to Palmer (Consolidated entity) $ 100,000
==========

This gain would, of course, increase consolidated retained earnings. The gain is identified with the issuer of the bonds, which is Palmer in this case. Therefore, the gain has no effect on noncontrolling (minority) interest.

126
Q

Bear Co. prepares its statement of cash flows using the indirect method. Bear sold equipment with a carrying value of $500,000 for cash of $400,000. How should Bear report the transaction in the operating and investing activities sections of its statement of cash flows?

Operating activities: $100,000 addition to net income; Investing activities: $400,000 cash inflow

Operating activities: $100,000 addition to net income; Investing activities: $500,000 cash inflow

Operating activities: $100,000 subtraction from net income; Investing activities: $500,000 cash inflow

Operating activities: $100,000 subtraction from net income; Investing activities: $400,000 cash inflow

A

Operating activities: $100,000 addition to net income; Investing activities: $400,000 cash inflow

The net loss ($400,000 – $500,000 = $(100,000)) would be removed (added back) from net income.

The cash from the sale would be included as a cash inflow from investing activities.

127
Q
A

$5,700,000

128
Q

During the current year, Ace Co. amortized a bond discount. Ace prepares its statement of cash flows using the indirect method. In which section of the statement should Ace report the amortization of the bond discount?

Investing activities

Financing activities

Operating activities

Supplemental disclosures

A

Operating activities

The amortization of the bond discount is classified as interest expense and has been deducted in arriving at net income. Using the indirect method, this amortization must be added to net income to compute net cash provided by operating activities.

129
Q
A
130
Q

Thyme, Inc. owns 16,000 of Sage Co.’s 20,000 outstanding common shares. The carrying value of Sage’s equity is $500,000. Sage subsequently issues an additional 5,000 previously unissued shares for $200,000 to an outside party that is unrelated to either Thyme or Sage. What is the total noncontrolling interest after the additional shares are issued?

$172,000

$252,000

$300,000

$140,000

A

$252,000

After Sage issued the additional $200,000 in equity, its total equity was $700,000 ($500,000 + $200,000).

Thyme now owns 16,000 shares out of the total 25,000 (20,000 + 5,000) shares outstanding, or 64% (16,000 ÷ 25,000).

The remaining 36% (100% − 64%) accounts for a noncontrolling interest of $252,000 ($700,000 × 36%).

131
Q

When entities face risk due to a lack of diversification, they must include disclosures in the notes to the financial statements about “vulnerability to concentrations.” Which of the following is not a necessary criterion of a “vulnerability to concentrations”?

The concentration exists at the date in the financial statements.

The concentration makes the entity vulnerable to the risk of a severe impact in the near term.

Severe impact events are those that are catastrophic.

There is a reasonable possibility that the events resulting in a severe impact related to the concentration will occur in the near term.

A

When firms lack diversification, they may be exposed to risks not faced by firms with adequate diversification. This is called “vulnerability to concentrations.” Such risks need to be disclosed in the notes to the financial statements when three conditions exist:

  1. Such a concentration exists at the date of the financial statements.
  2. The concentration makes the entity vulnerable to the risk of a near-term severe impact.
  3. It is at least reasonably possible that the events that could cause the severe impact will occur in the near term.

When management’s information indicates that these three conditions exist, the firm must disclose it is “vulnerable to concentrations.”

The answer choice “severe impact events are those that are catastrophic” is not one of the conditions necessary to require disclosure. While a concentrated event may be catastrophic, it only meets the disclosure requirement if it is also vulnerable to concentration.

132
Q

When firms lack diversification, they may be exposed to risks not faced by firms with adequate diversification. This is called “vulnerability to concentrations.” Such risks need to be disclosed in the notes to the financial statements when three conditions exist:

  1. Such a concentration exists at the ___of the financial statements.
  2. The concentration makes the entity ___to the risk of a near-term severe impact.
  3. It is at least ___possible that the events that could cause the severe impact will occur in the near term.

When management’s information indicates that these three conditions exist, the firm must disclose it is “___to concentrations.”

A

date

vulnerable

reasonably

vulnerable

133
Q

Which is the most appropriate financial statement to use to determine if a company obtained financing during a year by issuing debt or equity securities?

Statement of changes in stockholders’ equity

Statement of cash flows

Balance sheet

Income statement

A

Statement of cash flows

FASB ASC 230-10-45-1 requires cash flows to be presented in three categories in the statement of cash flows—operating, investing, and financing. Paragraph 6 states:

“A statement of cash flows should report the cash effects during a period of an enterprise’s operations, its investing transactions, and its financing transactions.” (Emphasis added)

Hence, one of the key purposes of the statement of cash flows is to disclose how a business financed its operations.

134
Q
A

225,000

135
Q

On April 30, Deer approved a plan to dispose of a segment of its business. For the period January 1 through April 30, the segment had revenues of $500,000 and expenses of $800,000. The assets of the segment were sold on October 15, at a loss for which no tax benefit is available. In its income statement for the calendar year, how should Deer report the segment’s operations from January 1 to April 30?

$300,000 reported as a net loss, as part of continuing operations

$500,000 and $800,000 included with revenues and expenses, respectively, as part of continuing operations

$300,000 reported as a loss from discontinued operations

$300,000 reported as an extraordinary loss

A

The operating loss and the loss on the sale must be reported as the loss for the discontinued operation. (Note: The concept of “extraordinary” items has been eliminated from GAAP and is therefore not a valid response.)

Revenues $ 500,000
 Expenses _(800,000)_
 Loss from discontinued operations $(300,000)
136
Q

Marble Co. prepared its statement of cash flows using the following amounts:

Net decrease in fixed assets $(3,750)
Depreciation expense 13,000
Gain on sale of equipment
(net book value, $3,250) 1,250
Capital expenditures 12,500

Marble reported net income of $20,000 at year-end. What amount should Marble report as net cash provided by operating activities?

$31,750

$33,000

$29,250

$19,500

A

$31,750

Given the limited information provided, the cash provided by operations can be found using the indirect method. The indirect method begins with net income and adjusts for noncash effects and changes in operating assets and operating liabilities, as follows:

Net income $20,000
Depreciation expense 13,000
Gain on sale of equipment (1,250)
Cash provided by operating activities $31,750

The decrease in fixed assets, capital expenditures, and proceeds from the sale of equipment would all be classified as investing activities in the statement of cash flows.

137
Q

Statement of Financial Accounting Concepts 8 (SFAC 8), Chapter 8, states that noncontractual asset disclosures could include all of the following except for:

the nature of the asset.

the quality of the asset.

the location of the asset.

the degree of nonperformance risk.

A

the degree of nonperformance risk.

SFAC 8 provides a summary of potential additional disclosures. For assets, the following items should be disclosed: the nature, quality, and location of the asset; future cash flows; relation to other line items; and significant contractual, statutory, regulatory, or judicial restrictions.

The degree of nonperformance risk of the asset is not one of the suggested disclosures.

138
Q

Which of the following items of stockholders’ equity can either be an increase or a decrease to total stockholders’ equity?

Paid-in capital in excess of par

Preferred stock (with par value)

Treasury stock

Accumulated other comprehensive income

A

Accumulated other comprehensive income

Accumulated other comprehensive income (AOCI) can either increase or decrease total stockholders’ equity because the elements of AOCI can be either cumulative gains or losses.

Paid-in capital in excess of par increases total stockholders’ equity. Treasury stock decreases total stockholders’ equity. Preferred stock (with par value) increases total stockholders’ equity.

139
Q

Town, Inc., is preparing its financial statements for the year ending December 31, 20X1. On December 1, 20X1, Town was awarded damages of $75,000 in a patent infringement suit it brought against a competitor. The defendant did not appeal the verdict, and payment was received in January 20X2, prior to the issuance of the financial statements. What is the reporting requirement?

Accrual only

Disclosure only

Both accrual and disclosure

Neither accrual nor disclosure

A

Both accrual and disclosure

Town, Inc., has a legally enforceable right to the settlement from the lawsuit on December 1, 20X1, so it would be reported in 20X1. The circumstances of the accrual of the gain should be disclosed in the interest of full disclosure.

140
Q

On July 1, 20X1, Dewey Co. signed a 20-year building lease that it reported as an operating lease. Dewey paid the monthly lease payments when due. How should Dewey report the effect of the lease payments in the financing activities section of its 20X1 statement of cash flows?

An inflow equal to the present value of future lease payments on July 1, 20X1, less 20X1 principal and interest payments

The lease payments should not be reported in the financing activities section.

An outflow equal to the 20X1 principal and interest payments on the lease

An outflow equal to the 20X1 principal payments only

A

The lease payments should not be reported in the financing activities section.

Cash outflows for operating activities per FASB ASC 230-10-45-17 include “cash payments to other suppliers or employees for other goods or services.” Operating lease payments are to appear in the operating activity cash flow section.

141
Q

Cash payments related to leases may be treated differently by lessees and lessors.

  1. For operating leases, cash flows appear in ___activities for both the lessee and the lessor.
  2. Lessees: For finance leases, the lessee reports the interest portion of the lease payment as cash flows from ___activities and the principal portion as cash flows from ___activities.
  3. Lessors: The lessor reports cash receipts from sales-type leases and direct financing leases as cash flows from ___activities.
A

operating

operating; financing

operating

142
Q
A

$(120,000)

143
Q
A

$1,240,000

In accounting for business combinations, the stockholders’ equity of the acquired entity is eliminated against the investment account. As a result, consolidated retained earnings include only the retained earnings of the parent company.

Thus, the Owen Corp. consolidated balance sheet on December 31, 20X1, would show a retained earnings amount of $1,240,000, an amount equal to Owen’s separate retained earnings.

144
Q
A

$1,314,600

145
Q
A

$201,900.

Items to be included in the revenue section of the 20X2 income statement:

Net sales revenue $187,000
Interest revenue 10,200
Gain on sale of equipment 4,700
Total revenues $201,900
========

Note: Generally accepted accounting principles require that the other items listed appear in other sections of the income statement or in another financial statement.

146
Q

Financial statement line item explanations which may require additional information for full disclosure purposes include all of the following except:

potential effect related to inability to perform or pay.

nature of primary activities.

line item relation to other line items.

degree of credit or nonperformance risk.

A

nature of primary activities.

Many financial statement line item explanations, such as local denomination demand deposits, do not require further explanatory information. However, other line items require varying degrees of disclosure. A summary of potential additional disclosures is as follows:

  • For assets: the nature, quality, and location; future cash flows; relation to other line items; and significant contractual, statutory, regulatory, or judicial restrictions.
  • For assets and liabilities resulting from financial instruments or other contracts: contractual or legal terms (e.g., timing of receipts and disbursements), degree of credit or nonperformance risk, potential effect related to inability to pay or perform, and method used to determine the cash flows.
  • Other disclosures could include equity instrument terms or conditions, potential effects of changing accounting methods, breakdown of aggregated line items, alternative measurements, and the line item’s relation to other line items.
147
Q

Glass Co. had net income of $70,000 during the year. Depreciation expense was $10,000. The following information is available:

Accounts receivable increase $20,000
Equipment gain on sale (sale price $100,000) 10,000 increase
Nontrade notes payable increase 50,000
Equipment purchases 40,000 increase
Accounts payable increase 30,000

What amount should Glass report as net cash provided by investing activities in its statement of cash flows for the year?

$50,000

$10,000

$60,000

$(40,000)

A

$60,000

Cash flows from investing activities involve the use and receipt related to nonoperating assets (i.e., property, plant, and equipment; equity and debt securities; notes receivable; etc.). Cash provided by investing activities is found in an identical manner under the direct or indirect method of preparing the statement of cash flows.

In this example, only two items would be classified as investing activities: the equipment purchase and equipment sale. The sale resulted in a $100,000 cash inflow and the purchase resulted in a $40,000 cash outflow, for a net cash provided amount $60,000 ($100,000 inflow − $40,000 outflow).

The change in accounts receivable and accounts payable balances would be captured in the operating section of the statement of cash flows. The change in notes payable would be captured in the financing section.

148
Q

Dove Inc. owns 100% of Flom Co. On January 2, 20X3, Dove sold equipment with an original cost of $120,000 and a carrying amount of $84,000 to Flom for $108,000. It is Dove’s policy to use straight-line depreciation with a useful life of 10 years for equipment like that sold to Flom. The equipment had no residual value. Flom is using straight-line depreciation over 6 years with no residual value. In Dove’s December 31, 20X3, consolidating worksheet, by what amount should depreciation expense be decreased?

$18,000

$6,000

$0

$12,000

A

$6,000

When dealing with unrealized gains or losses in a consolidated financial statement setting, the objective is to defer unrealized gains to establish both historical cost balances and recognize appropriate income within the consolidated financial statement.

The unrealized gain of the sale of the equipment to Flom is located in the cost of the equipment on Flom’s books.

Depreciation expense on a consolidated basis should be the depreciation that would have been expensed on Dove’s books if the equipment had not been sold.

Depreciation on Flom’s books (unrealized gain)
($108,000 ÷ 6) $18,000

Depreciation on Dove’s books (original cost)
($120,000 ÷ 10) 12,000
Difference $ 6,000

149
Q

The entire amount (100%) of any unrealized intercompany profit or loss should be eliminated for consolidation purposes T/F

A

True

150
Q

Which of the following disclosures should prospective financial statements include?

Summary of significant accounting policies

Both summary of significant accounting policies and summary of significant assumptions

Summary of significant assumptions

Neither summary of significant accounting policies nor summary of significant assumptions

A

Both summary of significant accounting policies and summary of significant assumptions

The AICPA’s “Statement of Standards for Accountants’ Services on Prospective Financial Information” governs the preparation of prospective financial statements

. It requires that accountants provide summaries of the significant accounting policies and the assumptions used to prepare these forward-looking statements.

The same full disclosure principle that guides the preparation of historical financial statements applies to the reporting of prospective financial statements.

151
Q

The following statements describe the form and content of accounting policy disclosure:

a. The disclosure encompasses important judgments as to ___of principles relating to recognition of revenue and allocation of asset costs to current and future periods.
b. The disclosure encompasses principles and methods that involve the selection from among existing acceptable ___.
c. Principles and methods peculiar to the __ are disclosed, even if the principles and methods are predominantly followed in that industry.
d. ___or innovative applications of generally accepted accounting principles are disclosed.
e. Policy disclosure is particularly useful if presented in a separate summary schedule, preceding the notes to the financial statements or as the first note. This should be appropriately identified (e.g., “Summary of Significant Accounting Policies”). T/F

f. Policy disclosure in a summary schedule should duplicate information presented elsewhere in the financial statements. T/F
g. In some cases, policy disclosure may need to be cross-referenced to information disclosed in other parts of the financial statements (e.g., other notes). T/F

A

appropriateness

alternatives

industry

Unusual

True

FALSE - Should NOT duplicate info presented elsewhere

True

152
Q

Estimates are a necessary part of the preparation of financial statements. T/F

A

True

153
Q

Disclosure of these significant estimates must be made when the following conditions are present:

a. It is at least reasonably possible that the estimate of the effect on the financial statements will ___in the near term due to one or more future confirming events (reasonably possible is a chance more than remote but less than likely).
b. The effect of the change would be ___.

A

change

material

154
Q

Where does the noncontrolling interest appear on the balance sheet?

In the owners’ equity section

In the liability section

Between the liability section and the owners’ equity section

None of the answer choices are appropriate disclosure of the noncontrolling interest on the balance sheet.

A

In the owners’ equity section

FASB ASC 810-10-45-16 requires that the noncontrolling interest be reported in the owners’ equity section.

155
Q

When the direct method of preparing a statement of cash flows is used, an enterprise should provide a reconciliation of net income to net cash flows from which activity?

Financing

No reconciliation should be provided.

Operating

Investing

A

Operating

A business enterprise that provides a set of financial statements intended to report financial position and results of operations must provide a statement of cash flows (SCF) for each period for which results of operations are presented.

The primary purpose of the SCF is to provide information about the cash receipts and cash payments of an enterprise during a period of time. The SCF is presented in three sections: cash flows from operating activities, cash flows from investing activities, and cash flows from financing activities (in that order).

The SCF can be prepared using the direct or indirect method. If the direct method is used, net income and net cash flows from operating activities must be reconciled.

156
Q

Rory Co.’s prepaid insurance was $50,000 at December 31, Year 2, and $25,000 at December 31, Year 1. Insurance expense was $20,000 for Year 2 and $15,000 for Year 1. What amount of cash disbursements for insurance would be reported in Rory’s Year 2 net cash flows from operating activities presented on a direct basis?

$45,000

$20,000

$30,000

$55,000

A

$45,000

The $45,000 amount is calculated as follows:

Insurance expense – Year 2 $20,000
Increase in prepaid insurance
($50,000 – $25,000) 25,000
Total $45,000

157
Q

A company reports the following information as of December 31:

Sales revenue $800,000
Cost of goods sold 600,000
Operating expenses 90,000
Unrealized holding gain on available-
for-sale debt securities, net of tax 30,000

What amount should the company report as comprehensive income as of December 31?

$30,000

$110,000

$200,000

$140,000

A

$140,000

158
Q
A

$260,000

159
Q

The following information was taken from Baxter Department Store’s financial statements:

Inventory at January 1 $ 100,000
Inventory at December 31 300,000
Net sales 2,000,000
Net purchases 700,000

What was Baxter’s inventory turnover for the year ending December 31?

2.5

5

10

3.5

A

2.5

Average inventory = (Beginning inventory + Ending inventory) ÷ 2:

  • ($100,000 + $300,000) ÷ 2 = $200,000

Inventory turnover = Cost of goods sold ÷ Average inventory:

  • ($100,000 + $700,000 - $300,000) ÷ (($100,000 + $300,000) ÷ 2) = 2.5
160
Q

What is the appropriate balance sheet classification for the portion of the principal balance owed on a 30-year mortgage note payable that will be paid in monthly installments during the next year?

Long-Term Liabilities

Stockholders’ Equity

Long-Term Investments

Current Liabilities

A

Current Liabilities

Current liabilities represent obligations whose liquidation is expected to require the use of current assets or the creation of other current liabilities and include obligations that have entered into the operating cycle (oftentimes, one year).

Monthly mortgage payments meet this definition because they will require the use of current assets to be settled (i.e., cash) and are due within the next operating cycle. Based on these factors, they should be reported as current liabilities in the balance sheet.

The other answer choices are not the appropriate balance sheet classifications for the upcoming mortgage payments. The payments are not stockholders’ equity because they are a repayment of debt, not a distribution to shareholders. They are not long term because they are due within one operating cycle.

161
Q

A company’s year-end comparative statement of financial position reflects the following changes from the prior year: cash increased by $40,000, total liabilities increased by $32,000, and all other assets decreased by $65,000. Which of the following statements is correct regarding the current-year change in the company’s stockholders’ equity?

It decreased by $32,000.

It decreased by $57,000.

It increased by $25,000.

It increased by $105,000.

A

It decreased by $57,000.

A statement of financial position (i.e., balance sheet) provides information about an enterprise’s assets, liabilities, and equity and their relationships to one another at a point in time. The statement delineates the enterprise’s resource structure (major classes and amounts of assets) and its financing structure (major classes and amounts of liabilities and equity).

For the statement, Assets = Liabilities + Equity; therefore, a net decrease of $25,000 ($40,000 – $65,000) in assets must equal a net decrease of $25,000 in liabilities and equity. If liabilities increased by $32,000, equity must have decreased by $57,000 ($32,000 + $25,000).

162
Q

King, Inc., owns 70% of Simmon Co.’s outstanding common stock. King’s liabilities total $450,000, and Simmon’s liabilities total $200,000. Included in Simmon’s financial statements is a $100,000 note payable to King. What amount of total liabilities should be reported in the consolidated financial statements?

$590,000

$550,000

$650,000

$520,000

A

$550,000

All intra-entity liabilities must be eliminated when preparing the consolidated financial statements:

King’s liabilities $450,000
Simmon’s liabilities 200,000
Intra-entity liability (100,000)
Consolidated total $550,000

163
Q

Statement of Financial Accounting Concepts 8 (SFAC 8), Chapter 8, provides descriptions and examples for all of the following areas that require consideration for note disclosure except:

non–entity specific information that is common knowledge.

the reporting entity.

financial statement line item explanations.

past events and current conditions that could impact an entity’s cash flow.

A

non–entity specific information that is common knowledge.

The FASB provides descriptions and examples for three areas that require consideration when preparing financial statement notes: financial statement line item explanations, information about the reporting entity, and information about past events and current conditions that could impact the entity’s cash flows but have not yet been incorporated into financial statement line items.

The FASB also provides a list of questions to be used as a tool when considering financial statement disclosures.

A “yes” response to a question means the FASB should consider disclosure but does not mean the FASB should require disclosure.

164
Q
A
165
Q

Which of the following would be added back to net income when reporting operating activities’ cash flows by the indirect method?

Excess of treasury stock acquisition cost over sales proceeds (cost method)

Both excess of treasury stock acquisition cost over sales proceeds (cost method) and bond discount amortization

Neither excess of treasury stock acquisition cost over sales proceeds (cost method) nor bond discount amortization

Bond discount amortization

A

Bond discount amortization

FASB ASC 230-10-55-1 describes the indirect method briefly as follows: “Given sufficiently detailed information, major classes of operating cash receipts and payments may be determined indirectly by adjusting revenue and expense amounts for the change during the period in related asset and liability accounts.”

Thus, bond discount amortization would be deducted from interest expense to compute the amount of cash paid for bond interest. The net effect would be to add this amortization amount to net income (as a noncash expense).

The excess of treasury stock acquisition cost over sales proceeds is not an income item; rather, the excess is an equity item debited to contributed capital from treasury stock transactions.

166
Q

In which of the financial statements can a user find details regarding items that caused retained earnings to increase and decrease during the fiscal year?

Income statement

Balance sheet

Statement of cash flows

Statement of changes in stockholders’ equity

A

Statement of changes in stockholders’ equity

The details showing the increases and decreases to the Retained Earnings account are reported in the statement of changes in stockholders’ equity.

Retained Earnings is also reported in the balance sheet, but the details of the items causing the changes to the beginning balance are only found in the statement of changes in stockholders’ equity.

167
Q

Which of the following items is included in the financing activities section of the statement of cash flows?

Cash effects of transactions involving making and collecting loans

Cash effects of transactions that enter into the determination of net income

Cash effects of transactions obtaining resources from owners and providing them with a return on their investment

Cash effects of acquiring and disposing of investments and property, plant, and equipment

A

Cash effects of transactions obtaining resources from owners and providing them with a return on their investment

Financing activities are associated with a company’s liabilities and stockholders’ equity. The list of financing activities in FASB ASC 230-10-45-14 and 45-15 therefore includes cash effects of transactions obtaining resources from owners and providing them with a return on their investment.

168
Q

Which of the following transactions is included in the operating activities section of a cash flow statement prepared using the indirect method?

Payment of principal on a long-term note payable

Purchase of treasury stock

Investment revenue recognized on an equity method investment

Purchase of property, plant, and equipment

A

Investment revenue recognized on an equity method investment

Investment revenue recognized on an equity method investment is included in net income. If an entity is using the indirect method, an adjustment to remove the investment revenue must be made to remove the effect from operating activities.

Cash paid for the purchase of treasury stock and cash paid for principal on a long-term note payable are included in financing activities. Purchase of property, plant, and equipment is included in investing activities.

169
Q

Birk Co. purchased 30% of Sled Co.’s outstanding common stock on December 31, 20X1, for $200,000. On that date, Sled’s stockholders’ equity was $500,000, and the fair value of its identifiable net assets was $600,000. On December 31, 20X1, what amount of goodwill should Birk attribute to this acquisition?

$50,000

$30,000

$20,000

$0

A

$20,000

When a company is bought, in whole or in part, the purchase price may exceed the fair values of all the company’s net assets. The amount of this excess is goodwill from the purchase.

Purchase cost of stock $200,000
Less 30% of identifiable assets (30% of $600,000) 180,000
Excess of purchase price over fair value
of assets (Goodwill) $ 20,000
========

170
Q
A

$1,665,000

171
Q
A

20X2: $91,000; 20X1: $(175,000)

172
Q

Babcock Company owes $50,000 to Mendenhall Corporation. Babcock also has an account receivable from Mendenhall of $45,000. Babcock wants to offset these items and report a net payable of $5,000 in their balance sheet. Which of the following is not required for Babcock to report these items in this manner?

Babcock has the right to set off the amount owed with the amount owed by the other party.

Babcock must have a signed agreement from Mendenhall to report the items in an offsetting manner.

The amounts owed by Babcock and Mendenhall to each other are clearly determinable.

Babcock’s right to offset the items is enforceable by law.

A

Babcock must have a signed agreement from Mendenhall to report the items in an offsetting manner.

Generally, the offsetting of assets and liabilities in the balance sheet is improper unless a right of setoff exists. The four criteria to establish a right of setoff that permits a firm to report items in an offsetting manner are:

each of the two parties owes the other determinable amounts,
the reporting party has the right to set off the amount owed with the amount owed by the other party,
the reporting entity intends to set off, and
the right of setoff is enforceable by law.
A signed agreement is the only answer choice that is not one of the four criteria.

173
Q

A partial listing of a company’s accounts is presented below:

Revenues $80,000
Operating expenses 50,000
Foreign currency translation adjustment gain,
net of tax 4,000
Income tax expense 10,000

What amount should the company report as net income?

$24,000

$30,000

$20,000

$34,000

A

$20,000

Revenues $80,000
Less: Operating expenses (50,000)
Less: Income tax expense (10,000)
Net income $20,000

Net income or loss for an accounting period is determined by matching realized revenues with those expenses and expired costs necessary to generate the related revenue. Revenues are inflows of assets or settlements of liabilities, during a period, from delivering or producing goods, rendering services, or other activities that constitute the entity’s ongoing major or central operations.

Expenses are outflows of assets or incurrences of liabilities, during a period, from delivering or producing goods, rendering services, or other activities that constitute the entity’s ongoing major or central operations, and include Operating Expenses and Income Tax Expense. Net income is $20,000 ($80,000 − $50,000 − $10,000).

Foreign currency translation adjustment gain is an item of Other Comprehensive Income (OCI), and is included in comprehensive income but not net income; OCI is reported as a direct charge or credit to equity.

174
Q

On September 1, 20X1, Canary Co. sold used equipment for a cash amount equaling its carrying amount for both book and tax purposes. On September 15, 20X1, Canary replaced the equipment by paying cash and signing a note payable for new equipment. The cash paid for the new equipment exceeded the cash received for the old equipment. How should these equipment transactions be reported in Canary’s 20X1 statement of cash flows?

Cash inflow equal to the cash received and a cash outflow equal to the cash paid and note payable

Cash inflow equal to the cash received and a cash outflow equal to the cash paid

Cash outflow equal to the cash paid less the cash received

Cash outflow equal to the cash paid and note payable less the cash received

A

Cash inflow equal to the cash received and a cash outflow equal to the cash paid

Included in cash inflows from investing activities per FASB ASC 230-10-45-12 are “receipts from sales of property, plant, and equipment and other productive assets.”

This pronouncement also includes, under the category of cash outflows from investing activities, “payments at the time of purchase or soon before or after purchase to acquire property, plant, and equipment and other productive assets.” (FASB ASC 230-10-45-13)

Canary Co. should treat the cash payments related to the equipment separately as both a cash inflow and cash outflow in the statement of cash flows.

175
Q

Which of the following assets or transactions is an element of comprehensive income?

Distributions to owners

Deferred revenue

Sales revenue

Investments by owners

A

Sales revenue

The statement of comprehensive income includes all revenues and expenses contained in the income statement/statement of profit and loss. Consequently, it would include sales revenue but would not include investments or distributions by owners or deferred revenue.

176
Q
A

$293,000

Under the principle of consolidation, the parent and subsidiary are considered a single economic entity. Thus, the consolidated balance sheet reports the combined (parent plus subsidiary) asset and liability accounts.

The single parent-sub entity owns all the net assets of both entities. Total stockholders’ equity accounts on the consolidated balance sheet equals the total stockholders’ equity of the parent plus the noncontrolling interest.

Therefore, Dallas, Inc., reports total stockholders’ equity account on December 31, 20X1, of $270,000 ($50,000 + $80,250 + $139,750) plus 20% of the total stockholders’ equity of Style of $23,000 ($20,000 + $44,000 + $51,000), which is $293,000.

177
Q

Which of the following transactions should be classified as investing activities on an entity’s statement of cash flows?

Issuance of common stock to the shareholders

Payment of cash dividend to the shareholders

Increase in accounts receivable

Sale of property, plant, and equipment

A

Sale of property, plant, and equipment

Investing activities involve asset transactions other than those related to operating results (e.g., sale of property).

178
Q

Which of the following must be included in a company’s summary of significant accounting policies in the notes to the financial statements?

Summary of long-term debt outstanding

Schedule of fixed assets

Revenue recognition policies

Description of current-year equity transactions

A

Revenue recognition policies

The purpose of the summary of significant accounting policies is to describe the accounting policies used by an entity. The revenue recognition policies therefore would be included in this note.

179
Q
A

$100,000

In a consolidated balance sheet, only the common stock of the parent entity is labeled “common stock.” Pare would report its own common stock ($100,000) on the December 31, 20X1, consolidated balance sheet.

Note: The 25% non-majority-owned common stock of Kidd Co. would be reported as part of “noncontrolling (minority) interest.”

180
Q

A company should recognize goodwill in its balance sheet at which of the following points?

Goodwill has been created in the purchase of a business.

The company expects a future benefit from the creation of goodwill.

The fair market value of the company’s assets exceeds the book value of the company’s assets.

Costs have been incurred in the development of goodwill.

A

Goodwill has been created in the purchase of a business.

The FASB only authorizes the recognition of goodwill in a purchase context.

181
Q

___ represents the reputation, business contacts, staff relationships, and industry experience that make a business more than a collection of assets.

A

Goodwill

182
Q

Which of the following statements regarding reporting for discontinued operations is incorrect?

The resulting gain or loss from discontinued operations should be reported net of applicable taxes.

In order for a sale to qualify as a discontinued operation, it needs to represent a strategic shift for the entity.

In order to qualify to be reported as a discontinued operation, the portion of the business being sold must have separate and identifiable cash flows.

A strategic shift is the sale of a product line that represents 25% or more of the entity’s total revenues.

A

A strategic shift is the sale of a product line that represents 25% or more of the entity’s total revenues..

In order for the sale of a portion of an entity to qualify for discontinued operations, it must represent a strategic shift for the entity, meaning that the sale will have a significant effect on the entity’s operations and financial results.

A strategic shift is the sale of a product line that represents 15% (not 25%) or more of the entity’s total revenues.

Discontinued operations are reported after income from continuing operations and therefore need to be reported net of tax.

The cash flows of the portion of the business to be sold must be clearly distinguishable from the other operations of the portion to be sold.

183
Q

A company has a 22% investment in another company that it accounts for using the equity method. Which of the following disclosures should be included in the company’s annual financial statements?

The company’s accounting policy for the investment

The reason for the company’s decision to invest in the investee company

Whether the investee company is involved in any litigation

The names and ownership percentages of the other stockholders in the investee company

A

The company’s accounting policy for the investment

Accounting policy disclosure includes the selection of accounting principles from existing acceptable methods. This would include the company’s use of the equity method.

The equity method must be used if the company has significant influence over the company whose stock has been acquired.

Generally, 20% ownership is evidence of significant influence, but it is possible that other factors would indicate otherwise. Consequently, the use of the equity is the selection of an accounting principle from existing alternatives (equity method or cost method).

184
Q

Which of the following statements regarding footnote disclosure related to “significant estimates” is incorrect?

Disclosures in the footnotes about significant estimates are encouraged but not required.

The disclosure about significant estimates needs to include the nature of the uncertainty.

Disclosure is required when the change in the estimate will have a material effect on the financial statements.

Disclosure is required when it is reasonably possible that the financial statement estimate will change in the near term.

A

Disclosures in the footnotes about significant estimates are encouraged but not required.

“Significant estimates” are estimates that affect the financial statements and for which there is a reasonable possibility that the estimate will change in the near term

. These significant estimates are also expected to have a material effect on the financial statements.

Disclosures regarding these estimates require an explanation of the nature of the uncertainty as well as specific language that it is reasonably possible that the estimate will change in the near term.

185
Q

Wand, Inc., has adopted FASB ASC 205-20 (Presentation of Financial Statements—Discontinued Operations). On October 1, 20X1, in a strategic shift, Wand, Inc., committed itself to a formal plan to sell its Kam division’s assets.

On that date, Wand estimated that the loss from the disposal of assets in February 20X2 would be $25,000. Wand also estimated that Kam would incur operating losses of $100,000 for the period of October 1, 20X1, through December 31, 20X1, and $50,000 for the period January 1, 20X2, through February 28, 20X2.

These estimates were materially correct. Assuming that the Kam division qualifies as a component, disregarding income taxes, what should Wand report as loss from discontinued operations in its comparative 20X1 and 20X2 income statements?

20X1: $0; 20X2: $175,000

20X1: $175,000; 20X2: $0

20X1: $125,000; 20X2: $50,000

20X1: $100,000; 20X2: $75,000

A

20X1: $125,000; 20X2: $50,000

ASB ASC 360-10-35-40 provides that when an entity is classified as held for sale, the unit must be written down to the fair value, so “a loss shall be recognized for any initial or subsequent write-down to fair value less cost to sell.”

Wand’s 20X1 loss from operations is $100,000 and the write-down to FMV is $25,000 and is reported in 20X1. The operating loss in 20X2 is $50,000, so Wand would report a $50,000 loss from discontinued operations before income taxes in 20X2.

186
Q

Polk Co. acquires a forklift from Quest Co. for $30,000. The terms require Polk to pay $3,000 down and finance the remaining $27,000. On March 1, Year 1, Polk pays the $3,000 down and accepted delivery of the forklift. Polk signed a note that requires Polk to pay principal payments of $1,000 per month for 27 months beginning July 1, Year 1. What amount should Polk report as an investing activity in the statement of cash flows for the year ended December 31, Year 1?

$9,000

$30,000

$3,000

$12,000

A

$3,000

Cash payments to purchase equipment are outflows from investing activities. The $3,000 down payment is an investing activity outflow.

The $27,000 financed and the principal payments are financing activities.

187
Q

Zest Co. owns 100% of Cinn, Inc. On January 2, 20X1, Zest sold equipment with an original cost of $80,000 and a carrying amount of $48,000 to Cinn for $72,000. Zest had been depreciating the equipment over a 5-year period using straight-line depreciation with no residual value. Cinn is using straight-line depreciation over three years with no residual value. In Zest’s December 31, 20X1, consolidating worksheet, by what amount should depreciation expense be decreased?

$16,000

$8,000

$24,000

$0

A

8000

When dealing with unrealized gains or losses in a consolidated financial statement setting, the objective is to defer unrealized gains to establish both historical cost balances and recognize appropriate income within the consolidated financial statement.

The unrealized gain of the sale of the equipment to Cinn is located in the cost of the equipment on Cinn’s books.

Depreciation expense on a consolidated basis should be the depreciation that would have been expensed on Zest’s books if the equipment had not been sold.

Depreciation on Cinn’s books (unrealized gain) (72,000 / 3) $24,000
Depreciation on Zest’s books (original cost) (80,000 / 5) 16,000
Difference $ 8,000

188
Q

Payne Co. prepares its statement of cash flows using the indirect method. Payne’s unamortized bond discount account decreased by $25,000 during the year. How should Payne report the change in unamortized bond discount in its statement of cash flows?

As an addition to net income in the operating activities section

As a financing cash outflow

As a financing cash inflow

As a subtraction from net income in the operating activities section

A

As an addition to net income in the operating activities section

The amortization of a bond discount is the difference between cash interest and interest expense. Cash paid for interest is reported in operating activities.

Amortization of a discount on bonds payable results in interest expense greater than cash interest. Because more expense has been deducted in computing income than the amount of cash paid for interest, the difference (captured in the change in the bond discount account) must be added to income to reconcile to the cash provided or used for operating activities.

189
Q
A

$356,000.

The net cash used in financing operations is $356,000, calculated as follows:

Payment for early retirement of long-term bonds $375,000
Dividend paid 31,000
Proceeds for sale of treasury stock (50,000)
Net cash used $356,000

There is no cash involved in the conversion of stock.

190
Q

Which of the following is a criterion that must be satisfied in order for an asset or a group of assets to be classified as “held for sale”?

The asset or group of assets is being actively marketed.

The asset or group of assets must be ready for immediate sale.

Management has committed to plan to sell the asset or group of assets.

All of the answer choices are criteria that must be satisfied for an asset or group of assets to be classified as held for sale.

A

All of the answer choices are criteria that must be satisfied for an asset or group of assets to be classified as held for sale.

There are six criteria that must be satisfied for an asset or group of assets to be classified as “held for sale.”

First, management has committed to plan to sell the asset or group of assets.

Second, the asset or group of assets must be ready for immediate sale.

Third, an active program to sell the asset or group of assets has been initiated.

Fourth, the sale of the assets or group of assets is probable and is to be completed within one year.

Fifth, the asset or group of assets is being actively marketed.

Sixth, it is unlikely that the plan to sell will be withdrawn.

Therefore, all of the answer choices are criteria that must be satisfied.

191
Q

Perez, Inc., owns 80% of Senior, Inc. During 20X1, Perez sold goods with a 40% gross profit to Senior. Senior sold all of these goods in 20X1. For 20X1 consolidated financial statements, how should the summation of Perez and Senior income statement items be adjusted?

Sales and cost of goods sold should be reduced by the intercompany sales.

Net income should be reduced by 80% of the gross profit on intercompany sales.

Sales and cost of goods sold should be reduced by 80% of the intercompany sales.

No adjustment is necessary.

A

Sales and cost of goods sold should be reduced by the intercompany sales.

FASB ASC 810-10-45-1 states: “In the preparation of consolidated financial statements, intra-entity balances and transactions should be eliminated. This includes…sales and purchases….Any intra-entity profit…shall be eliminated.”

The income statement adjustment process is greatly simplified because the goods sold to Senior were all subsequently sold to “outside” customers. This means that inventory will not require adjustment. The only adjustment needed is reduction of sales and cost of goods by the total dollar amount of the intercompany sales. Failure to do this would overstate those two items on the consolidated income statement.

192
Q

The primary purpose of a statement of cash flows is to provide relevant information about:

differences between net income and associated cash receipts and disbursements.

the cash receipts and cash disbursements of an enterprise during a period.

an enterprise’s ability to generate future positive net cash flows.

an enterprise’s ability to meet cash operating needs.

A

the cash receipts and cash disbursements of an enterprise during a period.

FASB ASC 230-10-10-1 contains standards for the financial accounting and reporting of an enterprise’s cash flows. This pronouncement notes: “The primary purpose of a statement of cash flows is to provide relevant information about the cash receipts and cash payments of an enterprise during a period.”

The information provided in the statement of cash flows must be used in conjunction with the related disclosures and other financial statements to assess differences between net income and associated cash receipts and disbursements, an enterprise’s ability to generate future positive net cash flows, and an enterprise’s ability to meet cash operating needs.

193
Q

The primary purpose of a statement of ___ ___is to provide information about the cash receipts and cash payments of an enterprise during a period of tim

A

Cash Flows

194
Q

Past events and current conditions potentially impacting the entity’s future cash flow include which of the following?

Related party transactions

Timing of asset cash flows

Subsequent events

Segment reporting

A

Subsequent events

Examples of past events and current conditions potentially impacting the entity’s future line items and cash flows but which have not yet been incorporated into financial statement line items include existing or potential litigation; suspected or known statute, judicial, regulatory, or contract violations; unrecognized existing commitments expected to be recognized in the future; events where significant uncertainty led to the decision to not recognize the event; and subsequent events.

Items not necessarily impacting line items that may require disclosure include dependency on one or a few customers or suppliers for profitability, input or output market volatility, uncertainty regarding an entity’s access to markets for inputs or outputs or ability to maintain a qualified workforce, and other significant specific entity risk.

Segment reporting and related party transactions are considered to be reporting entity disclosures, and the timing of asset cash flows is considered a financial statement line item explanation.

195
Q

On December 30, 20X1, Hale Corp. paid $400,000 cash and issued 80,000 shares of its $1 par value common stock to its unsecured creditors on a pro rata basis pursuant to a reorganization plan under Chapter 11 of the bankruptcy statutes.

Hale owed these unsecured creditors a total of $1,200,000. Hale’s common stock was trading at $1.25 per share on December 30, 20X1. As a result of this transaction, Hale’s total stockholder’s equity had a net increase of:

$1,200,000.

$100,000.

$80,000.

$800,000.

A

$800,000.

196
Q

Sun Co. is a wholly owned subsidiary of Star Co. Both companies have separate general ledgers and prepare separate financial statements. Sun requires stand-alone financial statements. Which of the following statements is correct?

After consolidation, the accounts of both Star and Sun should be combined together into one general-ledger accounting system for future ease in reporting.

After consolidation, the accounts of both Star and Sun should be changed to reflect the consolidated totals for future ease in reporting.

Consolidated financial statements should be prepared for both Star and Sun.

Consolidated financial statements (including the financial information of both corporations) should only be prepared by Star and not by Sun.

A

Consolidated financial statements (including the financial information of both corporations) should only be prepared by Star and not by Sun.

A company that controls another company must prepare consolidated financial statements. Star owns 100% of Sun and must consolidate the financial statements of the two companies.

197
Q
A

Vane should report $126,000, calculated as follows:

Net income before taxes ($600,000 – $420,000) $180,000
Income taxes ($180,000 x 0.30) 54,000
Net income from continuing operations $126,000

198
Q

Which of the following transactions is included in the operating activities section of a cash flow statement prepared using the indirect method?

Issuance of common stock to the shareholders

Payment of cash dividend to the shareholders

Gain on sale of plant asset

Sale of property, plant, and equipment

A

Gain on sale of plant asset

Gain on sale of a plant asset is included in net income. If an entity is using the indirect method, an adjustment to remove the gain on the sale of a plant asset must be made to remove the effect from investment activities.

Cash received from the sale of the plant asset is included in investing activities. Payment of a cash dividend and cash received from issuance of common stock to shareholders are included in financing activities.

199
Q

Paper Co. had net income of $70,000 during the year. Dividend payment was $10,000. The following information is available:

Mortgage repayment $20,000
Bonds payable–issued 50,000 increase
Inventory 40,000 increase
Accounts payable 30,000 decrease

What amount should Paper report as net cash provided by operating activities in its statement of cash flows for the year?

$30,000

$10,000

$20,000

$0

A

$0

Information provided supports the indirect method of computing net cash provided by operating activities. Only the increase in inventory and the decrease in accounts payable affect operating activities. The mortgage payment and the bonds issued are financing activities.

Net income $70,000
Inventory increase (40,000)
Accounts payable decrease (30,000)
Net cash provided by
operating activities $ 0

200
Q

Although future-oriented information requires considerable judgment and tends to not be disclosed, the FASB recommends in Statement of Financial Accounting Concepts 8 (SFAC 8) disclosure for the following two future-oriented types of information:

(1) Common knowledge and (2) management’s plans and strategies
(1) Estimates and assumptions and (2) management’s plans and strategies
(1) Terms and timing of cash flows and (2) nonperformance risk
(1) Estimates and assumptions and (2) alternative accounting methods

A

(1) Estimates and assumptions and (2) management’s plans and strategies

The Securities and Exchange Commission (SEC) provides protection for issuers regarding SEC-required information; however, that protection does not extend beyond SEC filings, potentially resulting in negative impacts (e.g., litigation) for entities providing information based upon predictions, projections, and forecasts about uncertain or unknown future events.

Therefore, the FASB does not require entities to disclose predictions of future outcomes that could result in negative consequences.

However, two types of forward-looking information are useful and should be provided: (1) estimates and assumptions, and (2) management’s existing plans and strategies for management-controlled matters.

201
Q

The FASB, when considering note disclosure, considers recognition, measurement, and presentation issues as well as other concepts such as objectives of financial reporting and qualitative characteristics of useful financial information. However, the FASB must also consider four constraints/limitations related to required information:

Relevance

Cost Constraint

Potential Adverse Consequences

Future-oriented information

A

Yep!

202
Q

The FASB, when considering note disclosure, considers recognition, measurement, and presentation issues as well as other concepts such as objectives of financial reporting and qualitative characteristics of useful financial information. However, the FASB must also consider four constraints/limitations related to required information:

(Which is missing?)

___

Const Constraint

Potential Adverse Consequences

Future-oriented information

A

Relevance

203
Q

Reporting entity disclosures include which of the following?

Asset nature, location, and quality

Related party reporting

Accounting method change impact

Potential litigation

A

Related party reporting

Many reporting entity disclosures are part of separate standards (e.g., segment reporting and related party information) and therefore do not require additional disclosure requirements.

Items included under this category include the nature of primary activities, special restrictions, advantages and disadvantages relative to other entities including unusual or unique regulatory or legal factors not readily available to users (e.g., one entity holds subsidiaries to sell and another integrates them into operations), related party disclosures, and disaggregated legal entity and segment information.

204
Q

One of the elements of a financial statement is comprehensive income. Comprehensive income excludes changes in equity resulting from which of the following?

Dividends paid to stockholders

Prior-period error correction

Loss from discontinued operations

Unrealized loss on investments in equity securities

A

Dividends paid to stockholders

Comprehensive income per SFAC 6, Elements in Financial Statements, encompasses all changes in equity of a business resulting from transactions with nonowners. Specifically: “It includes all changes in equity during a period except those resulting from investments by owners and distributions to owners.”

Based on this, dividends paid to stockholders would not be included in computation of comprehensive income.

205
Q

Burns Corp. had the following items:

Sales revenue $45,000
Loss on early extinguishment of bonds 36,000
Realized gain on sale of investment in equity securities 28,000
Unrealized holding loss on available-for-sale debt securities 17,000
Loss on write-down of inventory 3,100

Which of the following amounts would the statement of comprehensive income report as other comprehensive income or loss?

$28,100 other comprehensive loss

$11,000 other comprehensive income

$17,000 other comprehensive loss

$16,900 other comprehensive income

A

$17,000 other comprehensive loss

Other comprehensive income includes items such as gains and losses on foreign currency transactions designated as hedges, gains and losses on derivative instruments, gains or losses associated with pension or other postretirement benefits, and unrealized holding gains or losses on available-for-sale debt securities ($17,000).

All of the other items presented in the problem are included in net income.

206
Q

Mint Co.’s cash balance in its balance sheet is $1,300,000, of which $300,000 is identified as a compensating balance. In addition, Mint has classified cash of $250,000 that has been restricted for future expansion plans as “other assets.” Which of the following should Mint disclose in notes to its financial statements?

Restricted cash

Compensating balance

Both compensating balance and restricted cash

Neither compensating balance nor restricted cash

A

Both compensating balance and restricted cash

Material amounts of restricted cash must be segregated from “regular” cash for reporting purposes because it is not readily available for general use. Restricted cash is classified as current or long-term assets depending upon the date of availability for disbursement.

Compensating balances should be separately classified as being maintained as a compensating balance. It is classified as current or noncurrent based upon the terms of the agreement requiring the compensating balance, and details of the arrangement should be disclosed in the notes.

207
Q

Martin Co. had net income of $70,000 during the year. Depreciation expense was $10,000. The following information is available:

Accounts receivable increase $20,000
Equipment gain on sale increase 10,000
Nontrade notes payable increase 50,000
Prepaid insurance increase 40,000
Accounts payable increase 30,000

What amount should Martin report as net cash provided by operating activities in its statement of cash flows for the year?

$50,000

$100,000

$0

$40,000

A
208
Q

In Dart Co.’s Year 2 single-step income statement, as prepared by Dart’s controller, the section titled “Revenues” consisted of the following:

Sales $250,000
Purchase discounts 3,000
Recovery of accounts written off 10,000
Total revenues $263,000

In its Year 2 single-step income statement (statement of profit or loss), what amount should Dart report as total revenues?

$250,000

$260,000

$263,000

$253,000

A

$250,000

The single-step income statement presents all revenue and gains in the upper part of the statement. Purchase discounts are shown as deductions in the expense section.

Recovery of accounts written off has no effect on the income statement since cash is increased and allowance for doubtful accounts is decreased.

209
Q

New England Co. had cash provided by operating activities of $351,000; cash used by investing activities of $420,000; and cash provided by financing activities of $250,000. New England’s cash balance was $27,000 on January 1. During the year, there was a sale of land that resulted in a gain of $25,000 and proceeds of $40,000 were received from the sale. What was New England’s cash balance at the end of the year?

$27,000

$208,000

$248,000

$40,000

A

$208,000

210
Q

Tam Co. reported the following items in its year-end financial statements:

Capital expenditures $1,000,000
Sales-type lease payments 125,000
Income taxes paid 325,000
Dividends paid 200,000
Net interest payments 220,000

What amount should Tam report as supplemental disclosures in its statement of cash flows prepared using the indirect method?

$1,870,000

$745,000

$545,000

$1,125,000

A

Regardless of whether the direct or indirect method is used to determine cash flows from operating activities, the following items are required to be disclosed:

  • Amount of income taxes paid during the period ($325,000)
  • Amount of interest paid during the period ($220,000)

$325,000 + $220,000 = $545,000

211
Q

Barr Co. has total debt of $420,000 and stockholders’ equity of $700,000. Barr is seeking capital to fund an expansion. Barr is planning to issue an additional $300,000 in common stock, and is negotiating with a bank to borrow additional funds. The bank is requiring a debt-to-equity ratio of .75. What is the maximum additional amount Barr will be able to borrow?

$525,000

$330,000

$750,000

$225,000

A

$330,000

212
Q

When preparing a draft of its 20X1 balance sheet (statement of financial position), Mont, Inc., reported net assets totaling $875,000. Included in the asset section of the balance sheet were the following:

Treasury stock of Mont, Inc., at cost, which approximates
market value on December 31, 20X1 $24,000
Idle machinery 11,200
Cash surrender value of life insurance on corporate executives 13,700
Investments in equity securities 8,400

At what amount should Mont’s net assets be reported in the December 31, 20X1, balance sheet (statement of financial position)?

$851,000

$834,500

$850,100

$842,600

A

$851,000

Idle machinery, cash surrender value of life insurance on corporate executives, and investments in equity securities should appear in the asset section of the balance sheet (statement of financial position). Treasury stock should be presented as a reduction of stockholder’s equity, not as an asset.

Total net assets $875,000
Less Treasury stock 24,000
Net assets reported on balance sheet $851,000
========

213
Q

Wright Corp. has several subsidiaries that are included in its consolidated financial statements. In its December 31, 20X1, trial balance, Wright had the following intercompany balances before eliminations:

Debit Credit
Current receivable due from Main Co. 32,000
Noncurrent receivable from Main Co. 114,000
Cash advance to Corn Corp. 6,000
Cash advance from King Co. 15,000
Intercompany payable to King 101,000

In its December 31, 20X1, consolidated balance sheet, what amount should Wright report as intercompany receivables?

$146,000

$36,000

$152,000

$0

A

$0

Do not fall for this trick question. On a consolidated balance sheet no intercompany receivables (or payables) would appear. They would be eliminated in the consolidation process.

This rule is stated clearly in FASB ASC 810-10-45-1 as follows: “In the preparation of consolidated financial statements, intra-entity balances and transactions shall be eliminated.”

214
Q

Lando Company had the following items:

Service revenue $64,000
Gain on early extinguishment of bonds 41,000
Realized loss on sale of equipment 76,000
Foreign currency translation gain 23,000
Loss on impairment of building 9,500
Which of the following amounts would the statement of comprehensive income report as other comprehensive income or loss?

$53,000 other comprehensive loss

$42,500 other comprehensive income

$13,500 other comprehensive income

$23,000 other comprehensive income

A

$23,000 other comprehensive income

Other comprehensive income includes items such as gains and losses on foreign currency translation ($23,000), gains and losses on derivative instruments, gains or losses associated with pension or other postretirement benefits, and unrealized holding gains or losses on available-for-sale debt securities.

All of the other items presented in the problem are included in net income.

215
Q

Which of the following items should be shown as a component of comprehensive income?

Dividend paid to a shareholder

Foreign currency translation adjustment

Additional capital contribution

Deferred revenue

A

Foreign currency translation adjustment

The FASB requires that various “gains” and “losses” be excluded from the income statement and reported as direct charges or credits to equity; these items are included in comprehensive income, which includes all changes in equity during a period except those resulting from investments (i.e., additional capital contributions) by owners and distributions (i.e., dividends) to owners.

Deferred revenue is a liability and would not be reflected in equity until earned.

The only item listed that should be shown as a component of comprehensive income is a foreign currency translation adjustment.

216
Q

Rice Co. was incorporated on January 1, 20X1, with $500,000 from the issuance of stock and borrowed funds of $75,000. During the first year of operations, net income was $25,000.

On December 15, 20X1, Rice paid a $2,000 cash dividend. No additional activities affected owner’s equity in 20X1. On December 31, 20X1, Rice’s liabilities had increased to $94,000.

In Rice’s December 31, 20X1, balance sheet (statement of financial position), total assets should be reported at:

$617,000.

$598,000.

$692,000.

$600,000.

A

$617,000.

217
Q

When a parent-subsidiary relationship exists, consolidated financial statements are prepared in recognition of the accounting concept of:

materiality.

economic entity.

legal entity.

faithful representation.

A

economic entity.

FASB ASC 810-10-10-1 summarizes the purpose of consolidated statements as follows: “The purpose of consolidated statements is to present…the results of operations and the financial position of a parent and all its subsidiaries essentially as if the consolidated group were a single economic entity.”

Thus, consolidated statements reflect the economic entity concept.

218
Q

Dunbarn Co. had the following activities during the year:

Purchase of inventory $120,000
Purchase of equipment 80,000
Purchase of available-for-sale
debt securities 60,000
Purchase of treasury stock 70,000
Issuance of common stock 150,000

What amount should Dunbarn report as cash provided (used) by investing activities in its statement of cash flows for the year?

$(120,000)

$(140,000)

$(210,000)

$150,000

A

$(140,000)

A statement of cash flows reflects an entity’s cash receipts and cash payments classified by major uses (i.e., operating, investing, and financing activities). The investing activities section shows positive and negative cash flows for transactions involving assets that are not held for resale (i.e., inventory), such as investments in debt and equity securities; plant, property, and equipment; and intangible assets.

Dunbarn should report $(140,000) in investing activities related to the purchase (i.e., use of cash) of equipment and AFS (available for sale) debt securities. Inventory transactions are classified as operating activities; treasury stock and common stock transactions are classified as financing activities.

219
Q

Mr. Cord owns four corporations. Combined financial statements are being prepared for these corpora­tions, which have intercompany loans of $200,000 and intercompany profits of $500,000. What amount of these intercompany loans and profits should be included in the combined financial statements?

Intercompany loans, $200,000; Intercompany profits, $0

Intercompany loans, $200,000; Intercompany profits, $500,000

Intercompany loans, $0; Intercompany profits, $0

Intercompany loans, $0; Intercompany profits, $500,000

A

Intercompany loans, $0; Intercompany profits, $0

Intercompany loans and profits must be eliminated in the preparation of combined financial statements. This will result in balances of $0 in these accounts.

220
Q
A

$25,000

The amount reported on the consolidated statement retained earnings as “dividends paid” would include only dividends paid to majority shareholders directly, the $25,000 distributed by Pare Co.

Of the $5,000 dividends paid by Kidd, the parent’s share ($3,750) would be eliminated on the consolidated worksheet and the other $1,250 would be included in the noncontrolling (minority) interest.

However, the $1,250 would not be included in “dividends paid” on the consolidated statement of retained earnings.

221
Q

Host Co. has adopted FASB ASC 205-20 (Presentation of Financial Statements—Discontinued Operations). On October 1, 20X1, in a strategic shift, Host Co. approved a plan to dispose of a segment of its business. Host expected that the sale would occur on April 1, 20X2, at an estimated gain of $350,000. The segment had actual and estimated operating losses as follows:

01/01/X1 to 09/30/X1 $(300,000)
10/01/X1 to 12/31/X1 (200,000)
01/01/X2 to 03/31/X2 (400,000)

Assuming that the segment qualified as a component under FASB ASC 205-20-45, in its 20X1 income statement, what should Host report as a loss from operation of a discontinued segment?

$200,000

$600,000

$500,000

$250,000

A

$500,000

Under FASB ASC 205-20-45-3, the losses from a discontinued segment that qualifies as a component are reported in the period they occur. An anticipated loss on sale should be recognized by writing the component down to FMV. An anticipated gain on sale of the component should not be recognized until the day of the sale.

Loss 01/01/X1 to 09/30/X1 ($200,000)
Loss 10/01/X1 to 12/31/X1 ( 300,000)
Loss ($500,000)

222
Q
A

$2,110,000

.

223
Q

Mend Co. purchased a 3-month U.S. Treasury bill. Mend’s policy is to treat as cash equivalents all highly liquid investments with an original maturity of three months or less when purchased. How should this purchase be reported in Mend’s statement of cash flows?

As an outflow from operating activities

As an outflow from investing activities

Not reported

As an outflow from financing activities

A

Not reported

FASB ASC 230-10-45-1 states: “A statement of cash flows shall report the cash effect during a period of an entity’s operations, its investing transactions, and its financing transactions.”

It is further noted that these are the “same amounts as similarly titled line-items or subtotals shown in the statements of financial position as of those dates.” Since Mend’s policy is to treat these investments as cash equivalents, the purchase would not be reported in the statement of cash flows.

224
Q

Jane Co. owns 90% of the common stock of Dun Corp. and 100% of the common stock of Beech Corp. On December 30, Dun and Beech each declared a cash dividend of $100,000 for the current year. What is the total amount of dividends that should be reported in the December 31 consolidated financial statements of Jane and its subsidiaries, Dun and Beech?

$190,000

$100,000

$10,000

$200,000

A

$10,000

Intercompany dividends are eliminated in consolidation. The only dividends that remain after the eliminating entries are dividends paid to noncontrolling shareholders: 10% of Dun’s dividend of $100,000, or $10,000.

225
Q

Parent Co. owns 90% of the 10,000 outstanding shares of Subsidiary Co.’s common stock on December 31, year 1. On that date, the stockholders’ equity of Subsidiary was $150,000, consisting of $100,000 of no-par common stock and $50,000 of retained earnings. On January 2, year 2, Subsidiary issued 2,000 previously unissued shares for $24,000 to various outside investors. As a consequence of this transaction, Parent’s ownership share was reduced to 75%. Which of the following correctly reports this transaction?

The consolidated income statement reports a gain of $4,000.

The consolidated income statement reports a loss of $7,500.

Parent’s investment in Subsidiary is reduced by $4,500.

Parent’s investment in Subsidiary is increased by $3,000.

A

Parent’s investment in Subsidiary is reduced by $4,500.

226
Q

Karr, Inc., reported net income of $300,000 for 20X1. Changes occurred in several balance sheet accounts as follows:

Equipment $25,000 increase
Accumulated depreciation 40,000 increase
Note payable 30,000 increase

Additional Information

  • During 20X1, Karr sold equipment costing $25,000, with accumulated depreciation of $12,000, for a gain of $5,000.
  • In December 20X1, Karr purchased equipment costing $50,000 with $20,000 cash and a 12% note payable of $30,000.
  • Depreciation expense for the year was $52,000.

In Karr’s 20X1 statement of cash flows, net cash provided by operating activities should be:

$340,000.

$347,000.

$357,000.

$352,000.

A

$347,000.

Using the indirect method, Karr computes cash flow from operating activities as follows:

Reported 20X1 net income $300,000
Add depreciation expense 52,000
Deduct gain on sale of equipment ( 5,000)
Net Cash flow from operating activities $347,000
=========

Note: All items that are included in net income that do not affect net cash provided from, or used for, operating activities such as depreciation of property, plant, and equipment and amortization of finite-life intangible assets.

This includes all items whose cash effects are related to investing or financing cash flows, such as gains or losses on sales of property, plant, and equipment and discontinued operations (which relate to investing activities), and gains or losses on extinguishment of debt (which relate to financing activities). (FASB ASC 230-10-45-28)

227
Q

Palmyra Co. has net income of $11,000, a positive $1,000 net cumulative effect of a change in accounting principle, a $3,000 unrealized loss on available-for-sale debt securities, a positive $2,000 foreign currency translation adjustment, and a $6,000 increase in its common stock. What amount is Palmyra’s comprehensive income?

$10,000

$11,000

$17,000

$4,000

A

$10,000

Other comprehensive income includes foreign currency translation adjustments and unrealized holding gains or losses on available-for-sale debt securities. The change in accounting principle is an adjustment to retained earnings. The increase in common stock is not reflected in the income statement.

Net income $11,000
Unrealized loss on available-for-sale debt securities (3,000)
Foreign currency translation adjustment 2,000
Comprehensive income $10,000

228
Q

A company’s accounts receivable decreased from the beginning to the end of the year. In the company’s statement of cash flows (direct method), the cash collected from customers would be:

sales revenues plus accounts receivable at the beginning of the year.

the same as sales revenues.

sales revenues less the decrease in accounts receivable from the beginning to the end of the year.

sales revenues plus the decrease in accounts receivable from the beginning to the end of the year.

A

sales revenues plus the decrease in accounts receivable from the beginning to the end of the year.

Cash collected from customers includes:

  • sales revenue,
  • plus collections of accounts receivable from the prior year,
  • less recorded sales not yet received in cash.

A decrease in accounts receivable would indicate less collections than recorded sales not yet collected.

229
Q

When a full set of general purpose financial statements are presented, comprehensive income and its components should:

appear as a part of discontinued operations and cumulative effect of a change in accounting principle.

be reported net of related income tax effect, in total and individually.

appear in a supplemental schedule in the notes to the financial statements.

be displayed in a financial statement that has the same prominence as other financial statements.

A

be displayed in a financial statement that has the same prominence as other financial statements

The FASB requires that all items that are recognized as components of comprehensive income be reported in a financial statement that has the same prominence as other financial statements. However, the FASB does not prescribe a specific format for the display of such information.

230
Q

Bard Co. owned several subsidiaries at December 31. The following table shows each subsidiary’s total liabilities, excluding intercompany transactions, and percentage of stock owned by Bard:

Subsidiary__Total Liabilities__% Owned
Brock Co. $4,000,000 70
Harlson Co. 2,000,000 48
Porter Co. 7,000,000 80
Nortin Co. 5,000,000 100

What amount should Bard include as liabilities in its consolidated balance sheet at December 31?

$18,000,000

$16,000,000

$5,000,000

$12,000,000

A

$16,000,000

Intercompany balances in total are eliminated in the preparation of consolidated financial statements. Harlson is not included in the consolidated statements since Bard owns less than 50% of Harlson’s stock.

Brock Co. $4,000,000
Porter Co. 7,000,000
Nortin Co. 5,000,000
Total $16,000,000

231
Q

Articulation means that financial statements are:

separate and self-balancing.

unaffected by each other.

totally dependent on each other.

fundamentally interrelated.

A

fundamentally interrelated.

Articulation means that the elements of financial statements are fundamentally interrelated in two ways: (1) beginning balance + changes = ending balance, and (2) assets = liabilities + equity.

The concept of double-entry accounting (i.e., debits = credits) incorporates these relationships. In this way, financial statements show different aspects of the same transaction or event affecting the entity.

Financial statements are not separate and self-balancing—the balance sheet is dependent upon the current period income or loss from the income statement to be balanced.

Similarly, financial statements are affected by the other financial statements—changes in balance sheet elements (assets and liabilities) are reflected in the statement of cash flows.

232
Q

“___” is a term used to describe the interrelationship of the elements of the financial statements.

A

Articulation

233
Q

Articulation means that the elements of financial statements are fundamentally interrelated in two ways:

(1) beginning balance + changes = ending balance, and
(2) assets = liabilities + equity.

T/F

A

True

234
Q

mith Co. has adopted FASB ASC 205-20 (Presentation of Financial Statements—Discontinued Operations). On November 1, 20X1, in a strategic shift, Smith Co. contracted to dispose of an industry segment on February 28, 20X2. Throughout 20X1 the segment had operating losses. These losses were expected to continue until the segment’s disposition. Assuming that the segment qualifies as a component, if a loss is anticipated on final disposition, how much of the operating losses should be included in the loss on discontinued operations reported in Smith’s 20X1 income statements?

  1. Operating losses for the period January 1 to October 31, 20X1
  2. Operating losses for the period November 1 to December 31, 20X1
  3. Estimated operating losses for the period January 1 to February 28, 20X2

II and III only

I and II only

II only

I and III only

A

I and II only

The expected loss on the disposal to be reported in Smith Co.’s income statement for the period ended December 31, 20X1, would include actual operating losses for 20X1.

FASB ASC 205-20-45-3 states: “In a period in which a component of an entity either has been disposed of or is classified as held for sale, the income statement of a business entity or statement of activities of a not-for-profit entity (NFP) for current and prior periods shall report the results of operations of the component, including any gain or loss recognized in accordance with paragraphs 360-10-35-40 and 360-10-40-5, in discontinued operations.

The results of operations of a component classified as held for sale shall be reported in discontinued operations in the period(s) in which they occur

. The results of discontinued operations, less applicable income taxes (benefit), shall be reported as a separate component of income. For example, the results of discontinued operations may be reported in the income statement of a business entity as follows:

235
Q

The following information is from Mabel Co.’s year-end financial statements for the current and previous years:

Current YearPrevious Year
Prepaid expenses $ 10,000 $ 20,000
Accounts payable 50,000 30,000
Land 250,000 600,000

Land was sold during the current fiscal year for cash, resulting in a loss of $40,000. What is Mabel’s net adjustment to net income to determine net cash from operating activities?

$30,000

$70,000

$0

$(70,000)

A

$70,000

The reconciliation from net income to net cash flows from operating activities reconciles is required under the indirect method (as the operating section) or the direct method (as a reconciliation to the statement of cash flows).

It begins with net income; adjusts for non-cash expenses, losses, and gains in the income statement; and is further adjusted for the net change in current assets and current liabilities.

The reconciliation would add back the loss of $40,000 to net income, add $10,000 for the decrease in current assets (prepaid expenses) and add $20,000 for the increase in current liabilities (accounts payable) for a total positive adjustment of $70,000 ($40,000 + $10,000 + $20,000).

The decrease in land would be recorded in the investing section of the statement of cash flows for the amount actually received in cash.

236
Q

For a company to obtain a retail business license in a particular state, the company is required to pay the state the equivalent of three months of sales taxes on its projected retail sales. This amount is fully refundable after five years, provided the company has filed all required sales tax returns and paid all sales taxes due. Initially the company should report the payment related to this licensing requirement as:

a current asset.

a noncurrent asset.

an expense.

a noncurrent liability.

A

a noncurrent asset.

Current assets include cash or other assets expected to be turned into cash or consumed within the year. This deposit of sales taxes will not be returned within the year.

237
Q

A company’s year-end balance sheet is shown below:

Assets
Cash $ 300,000
Accounts receivable 350,000
Inventory 600,000
Property, plant, and equipment (net) 2,000,000
$3,250,000

Liabilities and Shareholders’ Equity
Current liabilities $ 700,000
Long-term liabilities 600,000
Common stock 800,000
Retained earnings 1,150,000
$3,250,000

What is the current ratio as of December 31?

  1. 43
  2. 67
  3. 79
  4. 93
A

The current ratio as of December 31 is 1.79:

Current ratio = Current assets / Current liabilities
= ($300,000 + $350,000 + $600,000) / $700,000
= 1.79

238
Q

In analyzing a company’s financial statements, which financial statement would a potential investor primarily use to assess the company’s liquidity and financial flexibility?

Balance sheet

Statement of cash flows

Income statement

Statement of retained earnings

A

Balance sheet

Evaluation of a company’s liquidity would necessitate computation of liquidity ratios such as the current ratio and acid-test ratio. Financial flexibility would be evaluated using debt and equity ratios.

The data used in computation of each of the above-mentioned ratios would be obtained from the balance sheet.

239
Q
A

$20,000

Baker should report $20,000 as net cash provided by operating activities:

Net income $78,000
Adjustments
Increase in accounts receivable ($82,000)
Increase in accounts payable 24,000 (58,000)
Net cash provided by operating activities $20,000

240
Q

Ace Co. issued 1,000 shares of its $10 par value common stock for $15 per share in cash. How should this transaction be reported in Ace’s statement of cash flows for the year of issuance?

$10,000 cash inflow from financing activities and $5,000 adjustment to arrive at cash flows from operating activities

$15,000 cash inflow from financing activities

$10,000 cash flow from investing activities and $5,000 adjustment to arrive at cash flows from operating activities

$15,000 cash flow from investing activities

A

$15,000 cash inflow from financing activities

Cash inflows from issuing a company’s own equity instruments such as common stock and preferred stock are reported as financing activities in the statement of cash flows. The full cash proceeds of $15,000 ($15 per share × $1,000 shares) would be reported as cash inflows from financing activities. The par value is not used because it does not impact the cash received.

241
Q

Which of the following cash flows per share should be reported in a statement of cash flows?

Both primary and fully diluted cash flows per share

Primary cash flows per share only

Fully diluted cash flows per share only

Cash flows per share should not be reported.

A

Cash flows per share should not be reported.

FASB ASC 230-10-45-3 states that “Financial statements shall not report an amount of cash flow per share.”

242
Q

On December 31, 20X1, in a strategic shift, Greer Co. entered into an agreement to sell its Hart segment’s assets. On that date, Greer estimated the gain from the disposition of the assets in 20X2 would be $700,000 and Hart’s 20X2 operating losses would be $200,000.

Hart’s actual operating losses were $300,000 in both 20X1 and 20X2, and the actual gain on disposition of Hart’s assets in 20X2 was $650,000.

Disregarding income taxes, what net gain (loss) should be reported for discontinued operations in Greer’s comparative 20X2 and 20X1 income statements?

20X2: $0; 20X1: $50,000

20X2: $(150,000) ; 20X1: $200,000

20X2: $50,000; 20X1: $(300,000)

20X2: $350,000; 20X1: $(300,000)

A

20X2: $350,000; 20X1: $(300,000)

243
Q

Which of the following transactions qualifies as a discontinued operation?

Phasing out of a production line representing 25% of the entity’s total revenue

Disposal of part of a line of business that represents 20% of the entity’s total revenue

All of the answer choices are discontinued operations.

Planned and approved sale of a segment

A

All of the answer choices are discontinued operations.

Discontinued operations result from a disposal that represents a strategic shift. Examples included in FASB ASC 205-20-55-83 to 55-101 include:

a sale of a product line that represents 15% of the entity’s total revenues;
a sale of a geographical area that represents 20% of the entity’s total assets;
a sale of all of one type of a reporting entity’s store formats that historically provided 30% to 40% of the reporting entity’s net income and 15% of the current-period net income;
the sale of an equity method investment representing 20% of the entity’s total assets; or
the sale of 80% of a product line representing 40% of total revenue, but only if the entity retains 20% of its ownership interest.
All of the disposals described in the answer choices are discontinued operations.

244
Q

Discontinued operations result from a disposal that represents a strategic shift. Examples included in FASB ASC 205-20-55-83 to 55-101 include:

  • a sale of a product line that represents 15% of the entity’s total revenues;
  • a sale of a geographical area that represents 20% of the entity’s total assets;
  • a sale of all of one type of a reporting entity’s store formats that historically provided 30% to 40% of the reporting entity’s net income and 15% of the current-period net income;
  • the sale of an equity method investment representing 20% of the entity’s total assets; or
  • the sale of 80% of a product line representing 40% of total revenue, but only if the entity retains 20% of its ownership interest.
A

Yep

245
Q

Which of the following is not disclosed on the statement of cash flows when prepared under the direct method, either on the face of the statement or in a separate schedule?

A reconciliation of net income to net cash flow from operations

A reconciliation of ending retained earnings to net cash flow from operations

The major classes of gross cash receipts and gross cash payments

The amount of income taxes paid

A

A reconciliation of ending retained earnings to net cash flow from operations

A reconciliation of ending retained earnings to net cash flow from operations would serve no useful purpose and is not required to be disclosed on the statement of cash flows. Each of the other items mentioned would be disclosed under the direct method.

246
Q

Penn Corp. paid $300,000 for the outstanding common stock of Star Co. At that time, Star had the following condensed balance sheet:

Carrying Amounts
Current assets $ 40,000
Plant and equipment (net) 380,000
Liabilities 200,000
Stockholders’ equity 220,000

The fair value of the plant and equipment was $60,000 more than its recorded carrying amount. The fair values and carrying amounts were equal for all other assets and liabilities. The combination is accounted for as an acquisition (initiated in a fiscal year beginning after December 15, 2008). What amount of goodwill, related to Star’s acquisition, should Penn report in its consolidated balance sheet?

$40,000

$60,000

$20,000

$80,000

A

$20,000

247
Q

Kline Co. had the following sales and accounts receivable balances at the end of the current year:

Cash sales $1,000,000
Net credit sales 3,000,000
Net accounts receivable, 1/1 100,000
Net accounts receivable, 12/31 400,000

What is Kline’s average collection period for its accounts receivable?

  1. 0 days
  2. 5 days
  3. 0 days
  4. 0 days
A

30.0 days

248
Q

Roro, Inc., paid $7,200 to renew its only insurance policy for three years on March 1, 20X1, the effective date of the policy. On March 31, 20X1, Roro’s unadjusted trial balance showed a balance of $300 for prepaid insurance and $7,200 for insurance expense. What amounts should be reported for prepaid insurance and insurance expense in Roro’s financial statements for the 3 months ending March 31, 20X1?

Prepaid insurance: $7,000; Insurance expense: $300

Prepaid insurance: $7,200; Insurance expense: $300

Prepaid insurance: $7,000; Insurance expense: $500

Prepaid insurance: $7,300; Insurance expense: $200

A
249
Q

____ ____is the excess of current assets over current liabilities.

A

Working capital

250
Q

Accumulated other comprehensive income is reported in which of the following financial statements?

The statement of financial position

The income statement

The statement of cash flows

The statement of comprehensive income

A

The statement of financial position

Other comprehensive is transferred to accumulated other comprehensive income each period.

Accumulated other comprehensive income is reported as part of equity on the balance sheet/statement of financial position.

251
Q

How should a gain from the sale of used equipment for cash be reported in a statement of cash flows using the indirect method?

In investment activities as a reduction of the cash inflow from the sale

In investment activities as a cash outflow

In operating activities as an addition to income

In operating activities as a deduction from income

A

In operating activities as a deduction from income

The cash proceeds from a sale of used equipment would be treated as a cash inflow from investing activities. Since these cash proceeds include both carrying value of the equipment and the gain from the sale, this gain would need to be deducted from income in order to avoid “double counting.”

252
Q

Parisian Company has bonds that have an original maturity of 10 years. The bonds will mature and be retired on May 25, Year 17. Parisian established a bond sinking fund, as required by the bond indenture (contract). The bond sinking fund will be used to retire the bonds when they mature in May of Year 17. How should the bonds be classified in the December 31, Year 16, balance sheet?

The bonds should be classified as a long-term liability.

Parisian can choose whether to classify the bonds as either current or noncurrent.

Because of the sinking fund, Parisian can eliminate both the bonds and the sinking fund from the balance sheet.

The bonds should be classified as a current liability.

A

The bonds should be classified as a long-term liability.

Even though the bonds will mature in less than one year, they are classified as long-term liabilities (not current maturities of long-term liabilities) because the cash to be used to pay off the bonds is restricted and has therefore been classified as a long-term asset.

Bonds in this situation should not be classified as current liabilities. Choice of classification (current vs. long-term) is not permitted. The presence of both the liability and the related asset (bond-sinking fund) are not offset on the balance sheet but are both reported in total.

253
Q

Which of the following defines equity as it relates to a business entity?

Total assets and liabilities

Net revenues

Total revenues less total expenses

Total assets less total liabilities

A

Total assets less total liabilities

Total assets less total liabilities defines equity. Equity (or net assets) is the residual interest in the assets of a business entity that remains after deducting its liabilities; equity represents the ownership interest.

Revenues are inflows of the assets or settlements of its liabilities from delivering or producing goods, rendering services, or other activities that constitute the enterprise’s ongoing major or central operations.

Expenses are outflows or other uses of assets or incurrences of liabilities from delivering or producing goods, rendering services, or carrying out other activities that constitute the enterprise’s ongoing major or central operations.

Revenues less expenses is net income (loss) and will increase (decrease) equity, but these accounts do not define equity.

254
Q

LLA, Inc. was capitalized through the issuance of 10,000 shares of $30 par common stock that was sold at $50 per share. LLA had net income as follows:

Year 1 $100,000
Year 2 200,000

If, during year 2, LLA paid dividends to its shareholders at $2.50 per share, what amount was LLA’s retained earnings balance and shareholders’ equity balance at the end of year 2?

Retained earnings $50,000; Shareholders’ equity $775,000

Retained earnings $275,000; Shareholders’ equity $775,000

Retained earnings $50,000; Shareholders’ equity $550,000

Retained earnings $275,000; Shareholders’ equity $550,000

A

Retained earnings $275,000; Shareholders’ equity $775,000

Shareholder’s equity recognizes equity (e.g., common stock and of preferred stock) and the accumulated earnings of the business. Retained earnings, a component of stockholders’ equity, is increased through net income and decreased through dividends and net losses.

Retained earnings in this example would be increased through year 1 and year 2 net income, $300,000, and decreased by the $2.50 per share dividend payment of $25,000, for an ending balance of $275,000 ($300,000 − $25,000).

Stockholders’ equity would have been initially recorded at the stock issuance value of $500,000 ($50* price per share × 10,000) shares. At the end of year 2 it would include the initial value and retained earnings, for a total balance of $775,000 ($500,000 common stock and additional paid-in capital + $275,000 retained earnings).

* On issuance, the $50 per share price would be split between common stock ($30 par value) and additional paid-in capital ($20 difference between par and fair value).

255
Q
A

$1,250,000.

256
Q

Which of the following items is included in accumulated other comprehensive income or loss?

A reduction of shareholders’ equity related to employee stock ownership plans

Unrealized holding gains or losses on investments in equity securities

Prior service costs not previously recognized as a component of net periodic pension costs

Unrealized gains and losses from the ineffective portion of a derivative properly designated as a cash flow hedge

A

Prior service costs not previously recognized as a component of net periodic pension costs

Accumulated other comprehensive income (AOCI) is a component of equity on the balance sheet, presented separately from retained earnings and additional paid-in-capital. Total items of other comprehensive income (OCI) are transferred to AOCI at the end of each reporting period. FASB defines OCI as “revenues, expenses, gains, and losses that under generally accepted accounting principles are included in comprehensive income but excluded from net income” (FASB ASC 220-10-20). These items have not yet been realized. Some examples of OCI include the following:

  • Foreign currency translation gains or losses
  • Gains and losses (effective portion only) on derivative instruments that qualify as cash flow hedges
  • Unrealized holding gains and losses on available-for-sale debt securities
  • Pension or postretirement gains or losses (not recognized immediately as a component of net periodic benefit cost)
  • Prior service costs or credits
257
Q

During 20X1, Pard Corp. sold goods to its 80%-owned subsidiary, Seed Corp. At December 31, 20X1, one-half of these goods were included in Seed’s ending inventory. Reported 20X1 selling expenses were $1,100,000 and $400,000 for Pard and Seed, respectively. Pard’s selling expenses included $50,000 in freight-out costs for goods sold to Seed. What amount of selling expenses should be reported in Pard’s 20X1 consolidated income statement?

$1,500,000

$1,475,000

$1,450,000

$1,480,000

A

$1,450,000

Since freight-out costs are paid by the seller (Pard), they are not included in the value of inventory by the buyer (Seed). Also, since they were paid on an intercompany sale, these costs should be eliminated from Pard’s consolidated income statement. Thus, consolidated selling expenses for 20X1 are:

Pard total - intercompany + Seed’s total
($1,100,000 - $50,000) + $400,000

$1,050,000 + $400,000 = $1,450,000

258
Q

Which of the following information should be disclosed in the summary of significant accounting policies?

Criteria for determining which investments are treated as cash equivalents

Refinancing of debt subsequent to the balance sheet date

Guarantees of indebtedness of others

Adequacy of pension plan assets relative to vested benefits

A

Criteria for determining which investments are treated as cash equivalents

FASB ASC 230-10-50-1, in a discussion of cash and cash equivalents, states: “An entity shall disclose its policy for determining which items are treated as cash equivalents.”

Note: The above requirement was embedded in a long paragraph discussing cash equivalents. The wording of the correct answer choice, particularly “criteria,” indicates a relation to policy, whereas none of the other answers imply such a relationship.

259
Q

A company decided to sell an unprofitable division of its business. The company can sell the entire operation for $800,000, and the buyer will assume all assets and liabilities of the operations. The tax rate is 30%. The assets and liabilities of the discontinued operation are as follows:

Buildings $5,000,000
Accumulated depreciation 3,000,000
Mortgage on buildings 1,100,000
Inventory 500,000
Accounts payable 600,000
Accounts receivable 200,000

What is the after-tax net loss on the disposal of the division?

$2,200,000

$200,000

$1,540,000

$140,000

A
260
Q

A company has the following accrual-basis balances at the end of its first year of operation:

Unearned consulting fees $ 2,000
Consulting fees receivable 3,500
Consulting fee revenue 25,000

The company’s cash-basis consulting revenue is what amount?

$26,500

$23,500

$30,500

$19,500

A

$23,500

The company’s cash-basis consulting revenue is $23,500:

Accrual basis consulting fee revenue $25,000
Unearned consulting fee–
cash received with no revenue 2,000
Consulting fees receivable–
revenue with no cash received (3,500)
Cash basis revenue $23,500

261
Q

Under the cash basis of accounting, revenue is recognized when cash is ___ and expenses are recognized when cash is ___.

Under the cash basis of accounting, No income or expense is accrued or deferred, and assets are not capitalized and subsequently depreciated. T/F

A

received; disbursed

True

262
Q

Which of the following should be disclosed as supplemental information in the statement of cash flows?

The issuance of stock to acquire a new warehouse

The amount of cash paid for taxes during the period

Both the issuance of stock to acquire a new warehouse and the amount of cash paid for taxes during the period

Neither the issuance of stock to acquire a new warehouse nor the amount of cash paid for taxes during the period

A

Both the issuance of stock to acquire a new warehouse and the amount of cash paid for taxes during the period

Information about all investing and financing activities of an enterprise during a period that affect recognized assets or liabilities, but that do not result in cash receipts or cash payments in the period (i.e., noncash transactions), shall be reported in related disclosures (FASB ASC 230-10-50-3).

The acquisition of assets for stock is an example of this type of item.

Disclosing the amount of cash paid during the period for interest and taxes is also required to be reported in related disclosures.

error_outline

263
Q

hich of the following is correct concerning financial statement disclosure of accounting policies?

The format and location of accounting policy disclosures are fixed by generally accepted accounting principles.

Disclosures should duplicate details disclosed elsewhere in the financial statements.

Disclosure of accounting policies is an integral part of the financial statements.

Disclosures should be limited to principles and methods peculiar to the industry in which the company operates.

A

Disclosure of accounting policies is an integral part of the financial statements.

All financial statement notes are an integral part of the financial statements.

264
Q

Numerous estimates are part of accrual accounting financial statements. Disclosures about these estimates are required in the notes to the financial statements. All but one of the following are correct statements about estimates disclosures; which statement is incorrect?

Determining whether the effect of a change in the useful life of an intangible asset will be material should be done either at the individual asset or major asset class level.

Certain significant estimates must be disclosed if there is a reasonable possibility that the estimate will change in the near term and the effect of the change will be material.

Financial statement preparers can reasonably assume that users of financial statements recognize that estimates are necessary in preparing financial statements and therefore no explicit statement regarding estimates is necessary.

GAAP requires an explanation that the preparation of the financial statements requires the use of estimates made by management in conformance with GAAP.

A

Financial statement preparers can reasonably assume that users of financial statements recognize that estimates are necessary in preparing financial statements and therefore no explicit statement regarding estimates is necessary.

GAAP requires the disclosure of the use of estimates in financial statements. Explicit communication in the notes to the financial statements about the use of estimates is necessary and should inform readers that many of the amounts reported are approximations, not exact amounts.

“Significant estimates” refers to estimates that have a reasonable possibility of changing in the near future and whose effect will be material—such items require disclosure in the notes to the financial statements. Whether disclosure is required for changes in intangible assets’ useful lives is based on determining if such changes will be material, and this determination needs to be done either at the individual asset or major asset class level.

265
Q

Qual Inc. purchased 90% of Saucer Co.’s outstanding common stock on December 31, 20X2, for $960,000. On that date, Saucer’s stockholders’ equity was $775,000, and the fair value of its identifiable net assets was $880,000. On December 31, 20X2, what amount of goodwill should Qual attribute to this acquisition?

$185000

$304000

$80000

$168000

A

$168000

When a company is bought, in whole or in part, the purchase price may exceed the fair values of all the company’s net assets. The amount of this excess is goodwill from the purchase.

Purchase cost of stock of $960,000 − 90% of identifiable assets (90% of $880,000) of $792000 = Excess of purchase price over fair value of assets (Goodwill) of $168000

266
Q

In a statement of cash flows, which of the following would increase reported cash flows from operating activities using the direct method (Ignore income tax considerations.)?

Dividends received from investments

Gain on early retirement of bonds

Change from straight-line to accelerated depreciation

Gain on sale of equipment

A

Dividends received from investments

FASB ASC 230-10-45-25 identifies several classes of receipts and payments that should be used in reporting cash flows from operating activities using the direct method. Included in the list of items are interest and dividends received.

The transactions or events related to the other items listed are not reported in the operating activities section.

267
Q

A company calculated the following data for the period:

Cash received from customers $25,000
Cash received from sale of equipment 1,000
Interest paid to bank on note 3,000
Cash paid to employees 8,000

What amount should the company report as net cash provided by operating activities in its statement of cash flows?

$15,000

$26,000

$18,000

$14,000

A

$14,000

268
Q

On July 4, 20X1, ABC, Inc., adopted a plan to terminate 100 employees effective September 1, 20X1. Each employee received a one-time termination benefit of $10,000 on September 4, 20X1. The termination was communicated to the employees on July 20, 20X1. Employees were not required to render service after August 1, 20X1, in order to receive the termination benefit. On what date should ABC record the cost of the termination benefit?

September 4, 20X1

July 4, 20X1

August 1, 20X1

July 20, 20X1

A

July 20, 20X1

FASB ASC 420-10-25-4 provides that the costs of one-time termination benefits be recognized and measured at its fair value on the communication date.

269
Q

If employees are entitled to receive termination benefits regardless of when they leave or if they will not be retained to render service beyond the minimum retention period, a liability for the termination benefits should be recognized at its fair value at the ___date

A

communication

270
Q

Contractual asset or liability disclosures identified in Statement of Financial Accounting Concepts 8 (SFAC 8), Chapter 8, include all of the following except:

legal terms.

degree of nonperformance risk.

method used to calculate the cash flow.

reporting segments.

A

reporting segments.

SFAC 8 provides a summary of potential additional disclosures for assets and liabilities resulting from financial instruments or other contracts: the contractual or legal terms (e.g., timing of receipts and disbursements), degree of credit or nonperformance risk, potential effect related to inability to pay or perform, and method used to determine the cash flows.

Reporting segments are not one of the suggested disclosure items.

271
Q

In preparing its cash flow statement for the year ending December 31, 20X1, Reve Co. collected the following data:

Gain on sale of equipment $ (6,000)
Proceeds from sale of equipment 10,000
Purchase of A.S., Inc., bonds
(par value $200,000) (180,000)
Amortization of bond discount 2,000
Dividends declared (45,000)
Dividends paid (38,000)
Proceeds from sale of treasury
stock (carrying amount $65,000) 75,000

In its December 31, 20X1, statement of cash flows, what amount should Reve report as net cash used in investing activities?

$176,000

$194,000

$170,000

$188,000

A

$170,000

Cash inflows from investing activities:
Proceeds from sale of equipment $ 10,000
Cash outflows for investing activities:
Purchase of A.S., Inc., bonds (180,000)
Net cash used in investing activities $170,000

272
Q
A

$151,400

The $151,400 amount is calculated as follows:

Net income $150,000
Increase in accounts receivable ($23,000 – $29,000) (6,000)
Increase in allowance for uncollectible accounts
($800 – $1,000) 200
Decrease in prepaid rent expense ($12,400 – $8,200) 4,200
Increase in accounts payable ($22,400 – $19,400) 3,000
Cash provided by operating activities $151,400

273
Q

What is the purpose of reporting comprehensive income?

To provide information for each segment of the business

To reconcile the difference between net income and cash flows provided from operating activities

To summarize all changes in equity from nonowner sources

To provide a consolidation of the income of the firm’s segments

A

To summarize all changes in equity from nonowner sources

SFAC 6 defines comprehensive income as “the change in equity [net assets] of a business enterprise during a period from transactions and other events and circumstances from nonowner sources. It includes all changes in equity during a period except those resulting from investments by owners and distributions to owners.”

274
Q

A company that uses the accrual method of accounting started the fiscal year with assets of $600,000 and liabilities of $400,000. During the fiscal year the company recorded credit sales of $250,000, of which $8,000 remained to be collected at year-end, and incurred expenses of $90,000, of which $72,000 was paid in cash.

A stock dividend valued at $10,000 was declared and issued to stockholders during the year. What is the year-end balance of total equity?

$360,000

$370,000

$350,000

$380,000

A

$360,000

Stockholders’ equity is equal to the net assets of the balance sheet (assets minus liabilities) and primarily includes retained earnings and stock issued and repurchased by the company.

Retained earnings, and therefore stockholders’ equity, is increased by net income and decreased by cash dividends declared. Stock dividends have no net impact on equity

. The beginning equity balance is equal to the net assets of $600,000 less liabilities of $400,000, or $200,000. Year-end equity is $360,000, computed as follows:

Beginning balance $200,000
Plus Credit sales 250,000*
Less Expenses (90,000)*
Ending balance $360,000
========
* Net income (revenues less expenses) is $160,000.

275
Q

Which of the following should be disclosed in a summary of significant accounting policies?

Concentration of credit risk of financial instruments

Adequacy of pension plan assets in relation to vested benefits

Basis of consolidation

Composition of plant assets

A

Basis of consolidation

When financial statements are issued purporting to present fairly the financial position, cash flows, and results of operations in accordance with generally accepted accounting principles, a description of all significant accounting policies of the reporting enterprise is required.

Examples of accounting principles and methods for which disclosure of policy is frequently made include, but are not limited to, depreciation methods, consolidation basis, interperiod tax allocation, inventory pricing, and revenue recognition methods.

The concentration of credit risk, composition of plant assets, and adequacy of pension plan assets are not disclosed in the summary of significant accounting policies.

276
Q

Which of the following information should be included in Melay, Inc.’s, 20X1 summary of significant accounting policies?

Property, plant, and equipment are recorded at cost with depreciation computed principally by the straight-line method.

Operating segment 20X1 sales are Alay $1M, Belay $2M, and Celay $3M.

Future common share dividends are expected to approximate 60% of earnings.

During 20X1, the Delay segment was sold.

A

Property, plant, and equipment are recorded at cost with depreciation computed principally by the straight-line method.

.FASB ASC 235-10-50-4 requires a description of all significant accounting policies when financial statements are issued. A listing of required policy disclosures by this pronouncement includes “basis of consolidation, depreciation methods, amortization of intangibles, inventory pricing” and several other items. Melay should include information concerning the cost and depreciation method(s) relating to property, plant, and equipment in its summary of significant accounting policies.

Note: While the other three information items should be disclosed in the financial statements, they should not be included in the summary of significant accounting policies:

During 20X1, the Delay segment was sold.
Operating segment 20X1 sales are Alay $1M, Belay $2M, and Celay $3M.
Future common share dividends are expected to approximate 60% of earnings.

277
Q

In preparing its cash flow statement for the year ending December 31, 20X1, Reve Co. collected the following data:

Gain on sale of equipment $ (6,000)
Proceeds from sale of equipment 10,000
Purchase of A.S., Inc., bonds
(par value $200,000) (180,000)
Amortization of bond discount 2,000
Dividends declared (45,000)
Dividends paid (38,000)
Proceeds from sale of Treasury
stock (carrying amount $65,000) 75,000

In its December 31, 20X1, statement of cash flows, what amount should Reve report as net cash provided by financing activities?

$27,000

$37,000

$20,000

$30,000

A

$37,000

Cash inflows from financing activities:
Proceeds from sale of treasury stock $75,000
Cash outflows from financing activities:
Dividends paid (38,000)
Net cash provided by financing activities $37,000

Dividends declared created a liability, but until they are paid, no cash flows out of the corporation.

278
Q

Reed Co.’s 20X1 statement of cash flows reported cash provided from operating activities of $400,000. For 20X1, depreciation of equipment was $190,000, impairment of goodwill was $5,000, and dividends paid on common stock were $100,000. In Reed’s 20X1 statement of cash flows, what amount was reported as net income?

$105,000

$305,000

$205,000

$595,000

A

$205,000

Dividends paid are reported as financing activities. The reconciliation of net income and cash provided by operating activities would reflect both of the other items as they are noncash expenses and losses.

  • Net income + Depreciation expense + Goodwill impairment loss = Cash provided by operating activities
  • X + $190,000 + $5,000 = $400,000
  • X = $205,000
279
Q

In a statement of cash flows, which of the following items is reported as a cash outflow from financing activities?

  1. Payments to retire mortgage notes
  2. Interest payments on mortgage notes
  3. Dividend payments

I and III

I, II, and III

I only

II and III

A

I and III

Cash outflows from financing activities include cash payments for dividends and principal payments to creditors. Interest payments are cash flows from operating activities.

280
Q
A

$770,000

Retained earnings on December 31, 20X1, would be computed as follows:

Total income since incorporation $1,570,000
Less cash dividends $(716,000)
Stock dividends (84,000) (800,000)
Retained earnings on December 31, 20X1 $ 770,000

Note: The excess of cost of treasury stock sold over cash received would be debited to Additional Paid-in Capital—Treasury Stock because the balance in that account is larger than the amount of the difference between the cost of the treasury stock sold and the cash received from its sale. The balance in Additional Paid-in Capital—Treasury Stock does not affect the retained earnings balance.

281
Q

Which of the following should not be disclosed in an enterprise’s statement of cash flows prepared using the indirect method?

Cash flow per share

Interest paid, net of amounts capitalized

Dividends paid on preferred stock

Income taxes paid

A

Cash flow per share

Cash flow per share should not be disclosed under either the direct or indirect method. FASB ASC 230-10-45-3 is very specific concerning per share cash flow disclosures: “Financial statements shall not report an amount of cash flow per share.”

282
Q

Dodd Co.’s debt securities at December 31 included available-for-sale securities with a cost basis of $24,000 and a fair value of $30,000. Dodd’s income tax rate was 20%. What amount of unrealized gain or loss should Dodd recognize in its income statement at December 31?

$6,000 gain

$4,800 gain

$0

$6,000 loss

A

$0

Available-for-sale debt securities are carried on the balance sheet at fair value. Unrealized changes in fair value between periods are reported in other comprehensive income for the period. Gains and losses are not reported on the income statement until realized. Items in other comprehensive income are reported net of their effective tax.

Dodd would recognize $0 in gain on its income statement. The unrealized gain in other comprehensive income is $4,800 ($6,000 gain × (1 – .20 tax rate)).

283
Q
A

$30,000

According to FASB ASC 810-10-20, a noncontrolling interest is “the portion of equity (net assets) in a subsidiary not attributable, directly or indirectly, to a parent. A noncontrolling interest is sometimes called a minority interest.”

The ownership interests in the subsidiary that are held by owners other than the parent is a noncontrolling interest. The noncontrolling interest in a subsidiary is part of the equity of the consolidated group.

The noncontrolling (minority) interest is the interest of third parties in the acquired company (Kidd).

Noncontrolling interest = Noncontrolling holding x Net assets of Kidd
= (1.00 - 0.75) x ($50,000 + $70,000)
= 0.25 x $120,000
= $30,000

Net assets can be computed in either of two ways: (1) book values of stockholders’ equity or (2) book value of assets less book value of liabilities. Here, the book values of stockholders’ equity are $50,000 for common stock and $70,000 for retained earnings.

284
Q

A noncontrolling interest is sometimes called a ___interest.”

A

minority

285
Q

Comprehensive income includes all of the following, except:

revenues from external customers.

interest expense to bondholders.

loss from a tornado.

dividends to shareholders.

A

dividends to shareholders.

Comprehensive income is defined in the FASB’s conceptual framework as “the change in equity (net assets) of a business enterprise, during a period, from transactions and other events and circumstances from nonowner sources.”

Comprehensive income includes all changes in equity during a period except those resulting from investments by owners and distributions to owners. Dividends are a distribution to owners and would therefore not be included in comprehensive income.

The other three answer choices (revenues from external customers, interest expense to bondholders, and loss from a tornado) would be included.

286
Q

Note section disclosures in the financial statements for pensions do not require inclusion of which of the following?

The company’s best estimate of contributions expected to be paid into the plan in the next fiscal year

The amount of net prior service cost or credit in accumulated other comprehensive income

The components of net period pension costs

A detailed description of the plan, including employee groups covered

A

A detailed description of the plan, including employee groups covered

The FASB requires extensive disclosures regarding pensions. Among these are the components of net periodic pension cost and the estimated plan contributions for the year following the latest year reported in the statement of financial position.

The FASB requires, for each annual statement of financial position presented, “the amounts in accumulated other comprehensive income that have not yet been recognized as components of net periodic benefit cost, showing separately the net gain or loss, net prior service cost or credit, and net transition asset or obligation” (FASB ASC 715-20-50-1).

Note that service cost is presented along with compensation expense, while the remaining items of net periodic pension cost are presented on a separate line item.

287
Q

A company has the following items on its year-end trial balance:

Net sales $500,000
Common stock 100,000
Insurance expense 75,000
Wages 50,000
Cost of goods sold 100,000
Cash 40,000
Accounts payable 25,000
Interest payable 20,000

What is the company’s gross profit?

$400,000

$275,000

$230,000

$500,000

A

$400,000

As presented on a multiple-step income statement, gross profit (or gross margin) is net sales less cost of goods sold. In this problem, gross profit is:

Net sales $500,000
Cost of goods sold (100,000)
Gross profit $400,000

288
Q

Bake Co.’s trial balance included the following at December 31, 20X1:

Accounts payable $ 80,000
Bonds payable, due 20X2 300,000
Discount on bonds payable 15,000
Deferred income tax liability 25,000

The deferred income tax liability is not related to an asset for financial accounting purposes and is expected to reverse in 20X2. What amount should be included in the current liability section of Bake’s December 31, 20X1, balance sheet (statement of financial position)?

$390,000

$420,000

$395,000

$365,000

A

$365,000

All deferred tax liabilities (and deferred tax assets) are classified as noncurrent. All of the remaining items listed are considered current as they are due and payable within the next year.

Accounts payable $ 80,000
Bonds payable 300,000
Discount on bonds payable (15,000)
Total $365,000

289
Q

Tulip Co. owns 100% of Daisy Co.’s outstanding common stock. Tulip’s cost of goods sold for the year totals $600,000 and Daisy’s cost of goods sold totals $400,000. During the year, Tulip sold inventory costing $60,000 to Daisy for $100,000. By the end of the year, all transferred inventory was sold to third parties. What amount should be reported as cost of goods sold in the consolidated statement of income?

$960,000

$1,000,000

$900,000

$940,000

A

$900,000

Under the acquisition method, a number of adjusting and eliminating entries are made during the consolidation process. Eliminations may be categorized as those related to the following:

  • The investment account
  • Current-year changes in the investment account
  • Year-end reciprocal balance sheet accounts
  • Reciprocal income statement accounts
  • Intercompany profits and losses

Since Daisy sold all of the inventory purchased from Tulip, Daisy would have recognized $100,000 in cost of goods sold (COGS). As Daisy is a 100%-owned subsidiary, 100% of the COGS from Tulip is eliminated (i.e., intercompany profits and losses). Total COGS on the consolidated statement of income is $900,000 ($600,000 + $400,000 − $100,000).

290
Q

Under the acquisition method, a number of adjusting and eliminating entries are made during the consolidation process. Eliminations may be categorized as those related to the following:

  • The ___account
  • ___-year changes in the investment account
  • Year-end ___balance sheet accounts
  • ___income statement accounts
  • ___profits and losses
A

Investment

Current-year

reciprocal

Reciprocal

Intercompany

291
Q

What are examples of reciprocal accounts?

A

AP & AR

Note Payable and Note receivable

Interest Income and Interest Expense

292
Q

According to the FASB conceptual framework, which of the following attributes would not be used to measure inventory?

Historical cost

Net realizable value

Present value of future cash flows

Replacement cost

A

Present value of future cash flows

SFAC 5, Recognition and Measurement in Financial Statements of Business Enterprises, discusses each of these measurement attributes. Each of the methods, except present value of future cash flows, would be acceptable in measuring inventory in certain circumstances. The use of present values of future cash flows was said to be useful in reporting long-term receivables.

Inventory is accounted for at lower of (historical) cost or market. Market is measured as replacement cost, net realizable value, or net realizable value less a normal profit margin.

293
Q

General purpose financial statements are directed toward the ___ ___ of various users and are feasible only because groups of users of financial information have similar needs.

___of financial statements: Financial statements individually and collectively contribute to meeting the objectives of financial reporting. Each financial statement provides a different kind of information and, generally, various kinds of information cannot be combined into a smaller number of financial statements without unduly complicating the information.

Classification and aggregation: Classification in financial statements facilitates analysis by grouping items with essentially ___characteristics and separating items with essentially different characteristics. Financial statements result from processing large amounts of data. Financial statements involve the need to simplify, condense, and aggregate information

Articulation of financial statements: Financial statements ____, or articulate, because they reflect different aspects of the same transactions or other events affecting the entity.

A

Common Interests

Usefulness

Similar

Interrelate

294
Q

A company acquired a building, paying a portion of the purchase price in cash and issuing a mortgage note payable to the seller for the balance. In a statement of cash flows, what amount is included in financ­ing activities for the transaction?

Cash payment

Acquisition price

Mortgage amount

Zero

A

Zero

The only cash involved in this transaction is the cash paid. It would be included in cash flows from investing activities.

295
Q

How should the amortization of bond discount on long-term debt be reported in a statement of cash flows prepared using the indirect method?

In operating activities as an addition to income

As a financing activities inflow

As a financing activities outflow

In operating activities as a deduction from income

A

In operating activities as an addition to income

Interest paid would be an expense included in the determination of net income, and therefore a cash outflow from operating activities. Amortization of bond discount is noncash interest expense. Net income must be increased by noncash expenses that did not result from a cash outflow.

296
Q
A

Total separate accounts receivable = $52,000 + $38,000 = $90,000
Less consolidated accounts receivable 78,000
Accounts receivable eliminated in consolidation $12,000

Intercompany receivables and payables are always eliminated in the consolidation process. Therefore, the $12,000 eliminated must represent the amount Shel owed to Pare for intercompany sales.

297
Q

On June 29, 20X4, Riff Inc. purchased all the issued and outstanding common stock of Jinks Co. for $2,640,000. Jinks had assets of $2,680,000 and liabilities of $540,000 on the acquisition date. Jinks’ recorded assets and liabilities had fair values of $2,800,000 and $620,000, respectively. In Riff’s June 30, 20X4, balance sheet, what amount should be reported as goodwill?

$80,000

$160,000

$500,000

$460,000

A
298
Q
A

$64,000

Total separate revenues = $400,000 + $280,000 = $680,000
Less consolidated revenues 616,000
Revenues eliminated in consolidation $ 64,000

Since intercompany sales are eliminated in the consolidation process, the $64,000 eliminated represents intercompany sales from Pare to Shel.

299
Q

For the 8 months ended August 31, year 5, the carpet division of a flooring company, which is considered a major line of business, had an operating loss of $115,000 from operations. On September 1, year 5, the board of directors voted to discontinue the division’s operations.

On December 31, year 5, the division was sold for a pretax loss of $135,000. The division’s operating loss for year 5 was $240,000.

The company’s income tax rate is 30%. What amount of loss should the company report as discontinued operations in the December 31, year 5, income statement?

$262,500

$182,000

$260,000

$168,000

A

Discontinued operations include all previously unrecognized gains or losses from the sale of the discontinued component and the results of operations for the discontinued component during the period, among other gains and losses.

During year 5, the division lost $240,000 and was sold for a loss of $135,000, creating a total gross loss of $375,000 ($240,000 + $135,000). On the income statement the total loss is reduced by the tax benefit associated with the loss of $112,500 ($375,000 × 30%) for a net loss of $262,500 ($375,000 − $112,500).

The $115,000 loss from the first 8 months of the year is already included in the $240,000 operating loss and is not needed to solve this problem.

300
Q

Which of the following is a component of other comprehensive income?

Changes in market value of inventory

Minimum accrual of vacation pay

Foreign currency-translation adjustments

Unrealized gain or loss on investment in equity securities

A

Foreign currency-translation adjustments

Some items included in comprehensive income include the following:

  • Foreign currency translation adjustments
  • Unrealized holding gains and losses that result from a debt security
  • Prior service costs or credits associated with pension or other postretirement benefits
301
Q

Comprehensive income comprises both of the following:

  • All components of net income
  • All components of other comprehensive income.

T/F

A

True

302
Q

Combined statements may be used to present the results of operations of:

commonly controlled companies.

neither commonly controlled companies nor companies under common management.

both commonly controlled companies and companies under common management.

companies under common management.

A

both commonly controlled companies and companies under common management.

FASB ASC 810-10-20 notes that in consolidation controlling financial interest resides with one of the companies included in the consolidation. In addition, combined financial statements would be useful where one individual owns a controlling financial interest in several entities that are related in their operations

Another use of combined statements is to present the financial position and the results of operations of entities under common management.

Thus, combined statements (not consolidated) may be used to present operating results of companies under common management, as well as commonly controlled companies.